to ppt january 15, 2015 d2

83
1. To what extent do the local government units of Marikina, Quezon City, Valenzuela and San Juan demonstrate participatory governance from 2010 to 2013? a. What are the major projects that the city have developed and implemented for the past 3 years ? b. What are the mechanisms that have been put into place to facilitate people participation? c. What role did constituents and NGOs play in the development and implementation of said projects? d. To what factors could the success or failure of projects be attributed to? e. How did the local government units address the contributory factors and problems that led to the failure/success of projects? 4.1 MARIKINA The major Projects That the City of Marikina has Develop and implemented for the past 3 Years. MARIKINA PROJECTS 2010 Participatory development is a process through which stakeholders can influenceand share control over development initiatives, and over the decisions andresources that affect themselves.(Ondrink, 2010). The Bicycle--friendly City Marikina City The shoe capital of the country is fast-becoming the biking capital as well. (Mangunay,2013). IN Marikina City, a fifth of the workforce will roll on two wheels and shear sweat power soon. That’s because up to 20% of Marikina City residents will be able to ride a bicycle to work when “The Bicycle- friendly City” program of Marikina is completed in 2010. Thanks to a novel idea thought up by the city government, which has introduced a 1

Upload: joshua-villanueva

Post on 27-Nov-2015

168 views

Category:

Documents


5 download

DESCRIPTION

joij

TRANSCRIPT

1. To what extent do the local government units of Marikina, Quezon City, Valenzuela and San Juan demonstrate participatory governance from 2010 to 2013?

a. What are the major projects that the city have developed and implemented for the past 3 years ?

b. What are the mechanisms that have been put into place to facilitate people participation?

c. What role did constituents and NGOs play in the development and implementation of said projects?

d. To what factors could the success or failure of projects be attributed to?

e. How did the local government units address the contributory factors and problems that led to the failure/success of projects?

4.1 MARIKINA

The major Projects That the City of Marikina has Develop and implemented for the past 3 Years.

MARIKINA PROJECTS 2010

Participatory development is a process through which stakeholders can influenceand share control over development initiatives, and over the decisions andresources that affect themselves.(Ondrink, 2010).

The Bicycle--friendly City Marikina City

The shoe capital of the country is fast-becoming the biking capital as well. (Mangunay,2013).

IN Marikina City, a fifth of the workforce will roll on two wheels and shear sweat power soon. That’s because up to 20% of Marikina City residents will be able to ride a bicycle to work when “The Bicycle-friendly City” program of Marikina is completed in 2010. Thanks to a novel idea thought up by the city government, which has introduced a cycling revolution of sorts since 1999 “Cycling is our answer to the soaring gas prices,” said Mayor Ma. Lourdes C. Fernando. “Bicycles are our provider of affordable mobility.” Aside from lower transport cost over short distances, cycling also reduces vehicle gas

emissions thereby leading to better health.

“Promoting Bicycle Use AmongMarikeños”

The main objective ways to increase the utilization of bicycles as an alternative mode of transport within the city. Focus was shifted from the bikeways of from infrastructure development towards behavioral change or actual use of bicycles.

This study would thus pioneer the need to examine the management of decentralization via schooling in HEIs that are subsidized by Philippine LGUs, to analyze the strategies developed in implementing

1

educational decentralization, to identify the difficulties LGUs have encountered in the management of the decentralization ofeducation in the tertiary level, and to draw policy implications for public administration and governance from the findings.

Participating in formulating the fundamental goals as well as in planning and carrying out an activity empowers stakeholders and fosters a sense of ownership. These facilitate effective project implementation, conscientious monitoring of activities, and sustainable outcomes.

Effective poverty reduction also requires greater flexibility in responding to problems and unexpected opportunities throughout project development, implementation, and monitoring.

Responsiveness and collaboration among intended beneficiaries, government, civil society, and the private sector at local, intermediate (district, province, etc.) and national levels promote social capital development and sound governance.

Marikina Eco Savers Project (2010)

The Eco-Savers Program is Marikina City’s innovative recycling scheme which promotes a culture of discipline among the youth through ecological solid waste management.The name of the program ‘eco-saver’ implies several meanings, which include the following: Saver of ecological system by being aware of sound environmental practices; An ecological solid waste management practitioner from a household; Economic savings realized through recycling of garbage.

Its main features and functioning are described in the BP description as follows:

Waste management at source. The program requires students to bring recyclable garbage from their respective households to school during an assigned Eco Day — the day when the garbage is going to be weighed and credited to their issued eco-savers passbooks. Students and parents are partners of the city government in practicing waste segregation and recycling practices at the household level.

Empowerment of elementary pupils in ecological solid waste management

even at an early age, the pupils at Marikina City’s public schools were given an opportunity to have meaningful involvement and create an influence to fellow youth in implementing sound ecological solid waste management practices by recovering recyclable materials from the household waste. This becomes a valuable experience that instils in them the values of discipline and concern for the environment.

Promotion of economic benefits from recyclables

This program was not only rewarding for the environment but also for the students who got points for the recyclables they bring to school. The recyclable garbage were valued according to the prevailing market price, i.e. PhP1.00 = 1 point. Points earned entitle the students to shop in the Eco-Savers bus, which carries educational materials such as dictionaries, books, school supplies and educational toys and basic commodities such as sugar, cocoa powder drink, and rice.

2

Incentive Scheme thru the Eco-savers passbooks

Each student issued the Eco-Savers passbook at the beginning of the school year. The passbook is where the points of the student-owner are credited and recorded according to the recyclable waste material he/she brings to school. An eco-saver who wishes to shop in the Eco-Savers Bus only needs to present this passbook, which entitles them to exchange their points with any of the goods sold inside.

Utilization of Eco-savers Bus

The Eco-Savers Bus or the Mobile Store is a converted city government facility which contains educational materials like dictionaries, story books, school supplies and educational toys and basic commodities such as sugar, cocoa powder drink, coffee and rice. This range of goods is exchangeable with the points secured by the students from their recyclables.

The advantages of this BP are easily understood:

In general, this is an educational awareness program on the ecological system among the constituents. It gives due focus on sound environmental practices for people from the household level to the schools to be ecological solid waste management practitioners. Ultimately, it seeks to instill the realization of economic savings through this waste management program. The Eco-Savers program is being implemented by the city’s Waste Management Office, in coordination with the Department of Education (DepEd). It was initially presented to DepEd in December 2003, and followed by a series of meetings that resulted in the formulation of the program’s several strategies.

Marikina Project Noah, Pagasa for city’s zero casualties

Marikina City praised the government project on disaster prevention and mitigation, and the weather bureau for the zero fatality and efficient rescue-and-relief operations in the city in the midst of Typhoon Maring.

Mayor Del de Guzman and Rep. Miro Quimbo attributed the timely, accurate and effective response of the local government unit (LGU) to the onslaught of Typhoon Maring to the precise warnings, substantial information and ample lead time provided by Project Nationwide Operational Assessment of Hazards (Noah) and the Philippine Atmospheric, Geophysical and Astronomical Services Administration (Pagasa) of the Department of Science and Technology (DOST).

Project Noah is a program of the national government as a response to the strict orders of President Aquino to put in place a responsive program for disaster risk reduction and mitigation that will provide lead-time warning to vulnerable communities against impending floods.

The project harnesses technologies and management services offered by the DOST through its agencies such as Pagasa, the Philippine Institute of Volcanology and Seismology, and Advanced Science and

3

Technology Institute, in partnership with the University of the Philippines (UP) National Institute of Geological Sciences and the UP College of Engineering.

“Marikina receives the brunt of rains and floodwaters from San Mateo, Antipolo and Montalban, which inundate us in such a speed that often result in fatalities and massive economic losses. In times of typhoons, the people and local officials essentially operate blindly in the absence of information on the amount of rainfall and estimated time of impact. We are either caught off-guard, which results in deaths and destruction of property like what happened with Typhoon Ondoy; or we overreact and conduct massive evacuation, which unnecessarily depletes the city’s resources and economically dislocates people,” Quimbo said.

But the Marikina solon said this has changed when the national government provided the LGUs with a potent tool to adjust and cope with typhoons through Project Noah and Pagasa. Through Project Noah, relocation of families in flood-prone areas has also become more efficient and systematic. It gives the officials and residents between three to six hours lead time to carry out an orderly evacuation.

De Guzman said, “We are not yet claiming victory, but we are hopeful that with this new tool given by the national government through Pagasa, we will survive this calamity without casualties.”

The city was able to accomplish its target of zero casualty in the face of the habagat (monsoon rains) in August 2012, which the city government credit to Project Noah.

The Marikina leaders also highlighted the benefits of a dynamic synergy between the national government and the LGUs. De Guzman and Quimbo said the national government and the LGUs had a shared vision of disaster prevention and mitigation, and this was realized when the President gave the local officials a potent tool through Project Noah.

“Disaster risk-reduction management will only be effective if you have proper tools and programs like Project Noah. Projects like these are worthy investments, as they give us the informations that are required in order to properly address the immediate needs of people during typhoons. In effect, the government can act more efficiently,” Quimbo said.

Project Noah was launched by the President last year in Balubad, Nangka, the most vulnerable area in Marikina.

The mechanisms that are put into place to facilitate people participation was to have a common goal cause with its people joined and participated. Other than the use of megaphones to further enhances the sounds which can also attract people to participate. Sometimes they chose to spread the word through social media just to participate because there you can attract more people to participate aside from having a wide range of area covered it is also cheap and more convenient. There are also times where they used the media televisions and radios to also spread the word it also has a wide range of

4

area but the difference is that it’s more expensive unlike the use of social where they can just post something then people would automatically notice.

How did the local government units address the contributory factors and problems that led to the success/failure of projects?

The local Government Units address the contributory factor and problems that led to the failure of projects Government failure is the public-sector analogy to market failure and occurs when government intervention causes a more inefficient allocation of goods and resources than would occur without that intervention. In not comparing realized inadequacies of market outcomes against those of potential interventions, one writer describes the of market failure as providing "only limited help in prescribing therapies for government success. The government's failure to intervene in a market failure that would result in a socially preferable mix of output is similarly referred to as passive government failure. Just as with market failures, there are different kinds of government failures that describe corresponding economic distortions.

Corruption is one of many tenacious problems associated with government failure. It is an example of the affinity of government —and life—with problems and failure. These are traceable to various causes or conditions, and include deficiencies and misdirection’s in group decision-making and in the disciplines of public administration and social science. Aside from corruption, there are other difficulties of a less morally reprehensible nature that are nonetheless wasteful and harmful and may contribute to corruption, poverty, inequality, and other “deep” problems. Issues of malfeasance deserve focal attention but should not overshadow those of misfeasance and nonfeasance.

Marikina City

I. Mekanismo ng Pakikilahok

1. Ipinaaalam at itinuturo ng pamahalaang lokal ang mga proyekto sa publiko.

Marikina launches project for small entrepreneurs

MARIKINA CITY, January 30 (PIA) -- In Marikina, ambulant vendors who comply with city regulations are provided financial assistance as additional capital for their livelihood.

Today, the city government has launched the “Tulong Pangkabuhayan Program.” The program aims to help the small entrepreneurs of the city, such as registered ambulant vendors, in their source of living.

Marikina City Mayor Del De Guzman said that having a job gives a man his honor.

“Even if it’s as simple as being a vendor, as long as it’s legal, we should be proud of it,” De Guzman stressed.

5

Lupong Tagapamayapa at Pang-kaayusan

Marikinas new bicycles to help maintain peace and order in the city

MARIKINA CITY, August 29 (PIA) -- Marikina Mayor Del De Guzman expressed his gratitude to the Philippine Seven Corporation and the Firefly Brigade for giving new bicycles to the city government through the Marikina City Bikeways Office (MCBO) Tuesday, August 28 at the Marikina City Hall Quadrangle.

These new bicycles are a big help to the city government in performing their responsibility to control and stop criminal acts around the city,†� De Guzman said.

In a release, the said bicycles shall be used by the Bike Patrol composed of employees from the MCBO, City Transportation Management and Development Office, and Office of the Public Safety and Security.

Aside from helping maintain peace and order in the city, the said group also patrols the 52-kilometer bike lanes in the city’s 16 barangays to inspect and apprehend traffic rule violators.

Here in Marikina, we promote bicycles instead of fuel-powered vehicles. Bicycling as alternative mode of transportation does not only benefit the environment but also a personâ health, De Guzman added. (Marikina-PIO/RJB/JEG-PIA-NCR)

Wednesday 29th of August 2012

4. Pagkakaroon ng "citizens volunteers" upang makilahok sa mga immersion programs.

Marikina leads training workshop for volunteers

The City Disaster Risk Reduction Management Council (CDRRMC) and the local government of Marikina conducted a three-day training workshop between March 19 and 21 to increase the effectiveness of volunteers and quality of volunteer service to citizens and communities.

The event, entitled, “Volunteer Management Systems Training Workshop,†� will be for disaster volunteers from different non-government organizations (NGOs).

The workshop also aims to discuss the definition and roles in volunteering, assess each NGOâ™s capacity in managing volunteers, and create an action plan in setting up volunteering management systems for Marikina.

Marikina City Mayor Del de Guzman said the workshop will be led by the CDRRMC, together with the non-profit organization Voluntary Service Overseas (VSO) Bahaginan, Metropolitan Manila Development Authority (MMDA), and the Office of Civil Defense (OCD).

6

Healthy partnership between the government and the private sector is important so we can save more lives during disaster operations. That is why both the government and the private sector must be trained and well-prepared, � De Guzman said.

Participants include Rusty Lopez Fire Rescue Volunteer, Marikina City Bikers, Marikina Filipino-Chinese Fire Brigade Volunteer, Philippine National Red Cross Marikina City Branch, Rover Scouts, MAGDALO Group, Kabalikat Civicom, SF Fire Brigade Volunteer, Rotary Club of Marikina, Lions Club International Marikina, Reservist Marikina Chapter, Metro Manila East Council-BSP and HAVI Logistics. ( EMVER CORTEZ Monday 18th of March 2013)

II. Direktang Pakikilahok

Ang pagbabantay ay mahalaga sa lipunan.

Marikina CCTV Camera

MANILA, Philippines – The city government of Marikina will installclosed-circuit television (CCTV) cameras in strategic locations to deter crimes and monitor the water level at the Marikina River.

Mayor Del de Guzman said the CCTV cameras could help save lives.

“These devices can help us in many ways. CCTVs can help prevent crimes and identify crime suspects,” De Guzman said.

He said a CCTV system would be installed under the Marikina Bridge in Barangay Sto. Niño to alert the residents, particularly those living in the riverside communities during heavy downpour.

City police chief Senior Superintendent Reynaldo Jagmis said majority of the CCTV cameras would be installed in crime-prone areas and major roads to monitor the flow of traffic.

De Guzman said the installation of the CCTV cameras would be completed by next month with the help of Marikina City Disaster Risk Reduction and Management Office and the Management Information System and Call Center.

7

4.2 QUEZON CITY

2010

1. Major projects of the city government of Quezon City concerning participatory governance from 2010 to 2013

How is local governance preformed in terms of participation of the people? To what extent is the their participation and contribution

2010: “Urban Farming”

- Encourages the cultivation of vegetables using organic methods to improve nutrition and as a source of livelihood. To date, 30 farms in public elementary schools, 37 community-based farms and 1 rooftop garden have been developed. The project has conducted 144 orientations with a total of 10,080 participants. Its partners include the Department of Agriculture, the QC Nutrition Council, SSDD, Barangays and the Division of City Schools. Barangay officials from as far as Amulong and Pangasinan have visited the Demo Farm located at the Quezon Memorial Circle. It has also hosted 30 school field trips from private schools.

(2010)

Mayor Herbert Bautista signed a memorandum of agreement (MOA) with Secretary Dinky Soliman of the Department of Social Welfare and Development (DSWD) and Representative Jorge “Bolet”Banal Jr., of the 3rd congressional district of Quezon City, for the development of an on-site socialized housing project for persons with disabilities (PWDs).

8

As part of a comprehensive approach, the QC government created the Task Force on Socialized Housing and Development of Blighted Areas pursuant to Local Executive Order No. 6, Series of 2010, for the transformation of blighted areas into decent communities.

The QC government also created a special task force, the STF Escopa, composed of its line agencies, the Office of the Representative of the 3rd Congressional District of Quezon City and the barangay councils of Escopa I, II, III and IV to help and facilitate the implementation of development projects in Escopa. The proposed development of on-site housing project for PWDs initiated by the city and the road improvement of the main roads of the four barangays is part of the overall development plan of Escopa. (http://www.quezoncity.gov.ph/index.php?option=com_content&view=article&id=1106:pwd-housing-district-3&catid=1:latest-news&Itemid=362)

(2012)

The first project is in Barangay Payatas, along Molave Avenue, near the Justice Cecilia Muñoz Palma High School. Of its 354 dwelling units, 192 have completed construction and 136 beneficiaries have moved in. Fifty-eight of the new owners are public school teachers, while 78 are informal settler families (ISFs). This is in partnership with the community and Habitat for Humanity.

The second is located in Barangay Kaligayahan, Novaliches, where 1,078 housing units are being built in an almost 4.9 hectare property. Ready for occupancy are 608 one-storey dwelling units with lofts, while the first 188 such units completed have been awarded to 47 teachers and 141 ISFs. This is in joint venture with property owner, Ofelia Arce, and developer, Phinma Property Holdings.

The third housing project is in Barangay Escopa, where Mayor Herbert Bautista has mobilized a Task Force that is resolving the decades-long problem of lack of Certificate of Entitlement to hundreds of families living in Escopa I, II, III and IV. The fourth housing project is along Vargas Lane in Barangay Culiat, about 160 meters from Visayas Avenue. Land development is being undertaken for the 9,200 sqm area, which has been designed to house 218 families, as well as a road network. Habitat for Humanity is also assisting here.

The fifth housing project is along Madja-as Road in Payatas. Land development is ongoing for the 11,364 sqm area, where 187 housing units will be constructed. The sixth housing project is in Chestnut Street, Fairview.

- The Quezon City government launched another housing project in a series of major housing projects on January 24, 2012. Special ceremonies were held for the official launching of Bistekville 2 at the project site. Mayor Herbert Bautista, Vice Mayor Joy Belmonte, members of the City Council and city officials will be joined by esteemed guest Vice President Jejomar Binay. Project partners Phinma Property Holdings and the Arce family were present in the event

9

2013: “Infra Structure Program”

- It is endorsed by the councilors to help with the rise of the newer building designed to improve the city’s structures.

The local government of Quezon City performance in terms of participation of the people the local Government of Quezon City shows good participation to the people they performed well by supporting and participating to Non-Government organization and establishing projects to help the people.

The active involvement of the QC residents in the legislative process was the primary reason of the 18th Quezon City Council in organizing a seminar entitled, Responsive Legislation through Active People’s Participation in barangays in QC.

The seminar being conducted in every barangay is primarily the avenue of the council members to reach out to the barangay level in aid of legislation.

As part of the stance of the council in promoting transparency and accountability, the seminar aims to deepen the understanding of the barangay officials, homeowners association, non-government organizations (NGOs), people’s organizations (POs) and other residents of the barangay in the familiarization of the legislative process being used in the council.

The latest innovations and reforms in the council were also tackled during the seminar including the basic orientation of the QC Council Website. Generally, this seminar is just one of the initiatives of the council members to urge the public to actively involve themselves in the legislative process and in order for their concerns to be addressed by the city council.

An example of the participation of the local government of Quezon City and Project establishment

Revised Quezon City Comprehensive Zoning Ordinance

The 18th Quezon City Council recently passed its Comprehensive Zoning Ordinance (Series of 2013) on March 04, 2013, which aimed to enhance the optimum use and allocation of land for future development needs of the inhabitants in QC.The said ordinance was approved after the extensive public consultations involving the different stakeholders from residential, commercial, industrial and institutional areas in QC that started last November 27 until December 12, 2012.

Disaster risk reduction summit for QC youth

Quezon City helps and participated with the people to teach the people mostly the youth on what to do in disaster. Youth summit with the theme, “Strengthening Youth Participation in Disaster Risk Reduction and Management” was recently held at the 14th floor conference hall of the QC Hall main building. The

10

summit aims to train and educate the young people of QC on how to respond during natural disasters such as typhoons, floods, earthquakes, and emergency situations like accidents.

Training programs were conducted in the future to teach the young people in other related areas such as first aid training and stress handling and management.

In compliance with UNICEF policies, the city government has been undertaking various efforts in disaster preparation by conducting trainings, seminars and workshops in different department, barangays, NGOs and NGAs to hone skills of volunteers, as well as citizens, in life-saving techniques.

Quezon City local Governance has a good performance in terms of participation and establishing projects to the people that’s what they been received a certificate of recognition from the Department of Interior and Local Government (DILG) for fostering a culture of excellence through the Local Government Performance Management System last December 17,2012 at the Performing Arts Theatre, Amoranto Sports Complex, Quezon City.

Participatory Electoral Platform/Agenda Building in Quezon City

Participatory governance started with platform building. Platform based election vs personality and money -oriented politics. Much better if the candidate’s platform is crafted by the people. They will have ownership in the platform. It became a covenant between the leaders and its constituents. The platform is based from the people’s experience, their aspirations. More importantly, public accountability is stronger specially when the candidate is already in office.

The platform centered on further improving the following key areas/sectors: Health and Nutrition, Education and Culture, Livelihood and Economic Development, Peace and Social Security and Sustainable Good Governance.

The main thrust was on how to create Jobs and livelihood, programs that will create Opportunities for all — Youth, women and all sectors. The main strategy for the platform/agenda to be realized is through Bayanihan sa Pamamahala.

There were series of community-based consultation-workshops in each 24 areas of Quezon City wherein the result of the workshop were presented back to the people for approval, adoption and endorsement( Ed Chavez, convenor, Task Force PAT, executive director, Center for Popular Empowerment (CPE), vice chair-internal, Council of Sectoral Representatives to the City Development Council,

25-B Matiyaga St. Barangay Pinyahan, Quezon City)

LOCAL SPECIAL BODIES:

• Local Health Board

Quezon City to offer affordable dialysis at Health Centers

11

The city health office opened dialysis centers to help kidney patients avail of the treatment for a minimal fee. This is part of the city government program to improve the delivery of health services to residents, especially to the city’s indigents who cannot afford the hospitalization and service costs.

The new treatment centers now have dialysis equipment and x-ray machines similar to the one at Toro Hills which currently provides affordable medical treatment to underprivileged renal patients.The 10 dialysis machines, some from donations from the United States, were strategically distributed to city health centers which was upgraded to be able to offer dialysis treatment.

City health office Chief Dr. Antonieta Inumerable said that the local government has intially hired several nurses to the four centers opened in Kamuning, Dona Narcisa, Novaliches and Escopa.

Local Tourism Board

CTAO was created primarily to initiate and implement the City Government's activities related to culture and tourism. It also acts on referrals/ instructions of the City Mayor on activities pertaining to culture, tourism, and historical matters; handles projects related to tourism; serves as a channel in the establishment and maintenance of sister-city relation both with local and international cities; supervises the registration of tourism oriented and tourism –related establishments in accordance with the provision of the QC Tourism Establishments Regulatory Ordinance of 1994

Local School Board

In view of the "no collection policy" of the Department of Education (DepEd), the QC government is setting aside some P1.42 million to help defray costs for graduation-related expenses. The 142 QC public schools were given P10,000 each covered the grant of the subsidy utilized by school administrators for repair or installation of stage, rental of sound system as well as chairs and tables, and procurement of decorations.

Mayor Herbert Bautista said that while the QC government supports DepEd's policy and its call for simple graduation rites, it also understands the predicament of schools that could not fully provide the needs of their students in making minimal but memorable commencement ceremonies.

A memorandum of agreement was signed by Mayor Bautista with the officers of the QC Public Elementary School Principals Association (QC-PESPA) and the QC Secondary Principals and Supervisors Association (QC-PRINSA) to formalize the grant of the subsidy, which will be taken out from the QC general fund.

It will be the second straight year that the QC government is providing the subsidy, which has been described by the mayor as the city's response to ease the financial burden of those parents who could

12

not afford the graduation expenses of their children.

QC has the largest number of public schools in the country up to this date where 96 elementary and 46 secondary public schools are spread over the city's 142 barangays.

All public elementary and high schools have been required to comply with the graduation policy set by Department of Education Order No. 9, series of 2013 to encourage austerity measures in schools, including wearing of school uniform during graduation rites.

• Peace and Order council

Quezon City Hall Compound

The Department of Public Order and Safety (DPOS) is tasked to:Provide public security, Augment auxiliary services on traffic management,Formulate plans and programs that improve public safety,Undertake protective and disaster relief services,Issue Temporary Terminal Permits for jeepneys and tricycles, traffic clearances for business establishments, security clearances for security agencies, and clearing of sidewalks of illegal vendors and obstructions.

The Local Government Code directs local governments to build meaningful participation of the citizenry in governance processes such as in local development planning and strengthen local development councils, both at the barangay and city level, and other special bodies.

The partnership between the Center for Popular Empowerment (CPE), an NGO focused in advocating participatory urban governance and the promotion of political, economic, and socio-cultural rights of the marginalized sectors in urban areas, and the city government of Quezon City began with a training on “Sustainable And Rights-Based Approach To Barangay Development in Selected Barangays in Quezon City” wherein CPE trained relevant staff in the area or barangay development.

The Center for Popular Empowerment (CPE) partnered with the Quezon City local government in the implementation of participatory development planning programs in four pilot barangays. The process called Barangay Development Planning through Participatory Learning and Action (BDP-PLA) is an approach which involves the community members in data gathering, problem analysis and prioritization of needs, formulation of community vision and mission, formulation of sectoral goals and objectives and preparation of the comprehensive 5-year development plan, annual investment plan and annual operational plan.

The process took 5-7 days of tedious process of workshops and focus group discussions which involved 40-70 participants from the communities and around 15-30 facilitators and documenters who are provides their respective expertise on development planning. This relies on the premise that with thorough planning and participation from the locals, the development plan of the government is representative of the ideals and needs of the people.

13

CPE then provided training to various units and offices of the Quezon City government who later on became local community facilitators and documenters in conducting the program.

The long-term objective of the process is in line with the Quezon City’s Vision of Sustainable Barangays and Participative governance wherein the process of development planning emanates from the barangays. (EXCERPTS FROM Overview of BDP-PLA in Quezon City by Edwin Chavez, Executive Director, Center for Popular Empowerment CPE)

In social preparation , before the actual conduct of BDP-PLA in the barangays, identified facilitators were trained to facilitate and document the workshop process. Community orientations were also held to level-off on the objectives and processes undertaken. These community sessions were also intended to gather the support of the community members, especially the members of the sangguniang barangay.

In data gathering, the community members were grouped into different teams to conduct data gathering simultaneously. They were guided by trained facilitators in data gathering and analysis. PLA tools are used in gathering information. Problems and opportunities are identified in this stage. Data gathering usually takes 1 to 2 days. In problem identification and prioritization , community planners used tools such as the “Problem Tree” or “Cause-Effect” diagrams in analyzing problems. This is done in each sectoral development sector. The problem trees were translated into “Needs Tree” which eventually used in identifying “Sectoral Goals and Objectives.”

Prioritizing needs is aided by the use of PLA tools such as “Matrix Ranking” or “Pair Wise Ranking Exercises”. Through the ranking exercises, the community members debated and brought into consensus perceived priorities through a set of criteria (i.e. urgency, scope, internal capacity to address the problems, availability of internal resources, etc.).In cross sectoral validation, the different sectors gathered into a plenary and presented the output for scrutiny of other groups. The groups provided feedback, asked clarifications, gave suggestions or recommendations to other groups. This process enabled feedback-giving among the groups and validation of data. Community Vision and Mission

The plenary group formulates vision and mission goals through a participatory process, thus increasing people’s ownership of the barangay’s vision and mission statements. These were done through the use of visual materials or through small workshops. After goals and objectives were identified, the community planners identified specific programs and projects to carry out in the next 3 to 5 years, together with the key result areas and target budget needed.

Plan Implementation, Monitoring and Evaluation

The Barangay Development Council is the main responsible body in implementing, monitoring and evaluating the programs and projects identified in the plan. The BDC members should thus be equipped with the necessary skills, knowledge and attitudes in program implementation, monitoring and evaluation.

14

Integrative, comprehensive and multi-sectoral approach

The composite team decided that the framework to be used in barangay planning shall be the framework used in city development planning.

The vision, mission and development priorities of the city were also reviewed during the training to level-off and integrate the elements in the barangay development plans to the wider framework which is the city framework.

The primary issue was on how to sustain and mainstream the initiative of participatory development planning in the city level. There are already trained staff from the city government, and NGOs who may carry on the technology.

Participation of the community members seem to be a problem during the first days of the barangay level planning session. But after after the frist day of the workshop, other community members seem to be interested in the process and eventually participated in the different sectoral workshops.

Another thing that was seriously considered, the strengthening of the Barangay Development Councils. The BDCs are the ones primary tasked in successfully implementing the development programs of the barangay.

There is also a serious consideration on how to mainstream the participatory local governance process of development planning in the city development planning process. Initially, the city government considered to expand the process to another 12 barangays . Doing this would entail administrative and logistical arrangements within the city government in order to effectively deploy key personnel and coordinate the activities. In other areas where BDP-PLA process has been implemented in all the barangays, the municipal LGU formed a Task Force or a Technical Working Group whose main task is to implement the participatory development planning process.

Bringing the people in governance is a major task that has still to be pursued. In Quezon City, this process has been started. Hence the importance of strong principled partnership between the LGUs and civil society organizations in achieving this vision.

The barangay development planning through PLA is just part of the whole framework of participatory governance—which has its form and substance to be observed. Oftentimes, participatory governance is

15

seen only through its form – forms of people’s participation in decision-making process i.e. consultation. But the quality of participation must also be put into serious consideration. Participation is empowering only if those who participate makes the decision and choices.

Participatory governance remains to be a challenge after more than 13 years of LGC implementation. It remains to be a vision to be realized in the majority of the local government units in the Philippines. In the National Capital Region, only Quezon City has embarked in participatory barangay development planning process. The 4 pilot barangays in Quezon City may serve as a jumping board in pursuing participatory development planning process and participatory governance in general, in the whole city. Lessons culled from this initial experience were further put the city to become a more quality city—wherein people in the communities have exercised their active citizenship.

PARTICIPATORY MECHANISM

LOCAL TOURISM BOARD

As Quezon City maneuvers in becoming a tourist destination in the Philippines, the 18th City Council recognizes the role of a good policy framework that facilitated tourism in achieving socio-economic growth in the City. After its final reading last September 17, 2012, the Quezon City Tourism Department Ordinance is now on its final phase in implementing it.

This ordinance is regarded as the pillar in creating investment, foreign exchange, and employment and to continue in molding an enhanced sense of national pride of all Filipinos particularly the residents of Quezon City.

City Councilors guarantee that, through this ordinance, local tourism development will gear towards the conservation and promotion of Filipino heritage, the development of national identity and the formation of civic pride.

Developed and functional tourism initiatives brought by the Tourism Department will surely bring more jobs for the City’s residents, better standards of tourism related establishments in generating more income for the government that ensure the improvement of the quality of life of its residents.

16

QUEZON CITY

I. Mekanismo ng Pakikilahok (PARTICIPATORY MECHANISMS)

2. May mataas na antas na bisyon at pagtugon ng pamahalaang lokal upang magkaroon ng tamang institusyon

Vision: The Quezon City Government envisions itself to be a model of effective governance and responsible leadership, working in partnership with the citizenry in building a Quality Community

Mission: To provide quality service which will make Quezon City an ideal community where people live, work and do business in a hospitable, progressive and peaceful environment

http://www.quezoncity.gov.ph/index.php?option=com_content&view=article&id=339:visionmission&catid=90

3. Pagpapaunlad ng pamahalaang lokal na magkaroon ng ugnayan sa pamamahala sa pamamagitan ng mga reporma ng Local Special Bodies (LSB) tulad ng:

a) Lupong Pangkalusugan (Health Board)

Construction of new Quezon City Hospital

The construction of the Batasan Hospital will be the Bautista Administration's top priority for this term. Most of the hospitals and medical establishments like East Avenue Medical Center, Philippine Heart Center, Lung Center of the Philippines, National Kidney and Transplant Institute, Philippine Children's Medical Center and Veterans Memorial Medical Center are located kilometers away from the barangays in the city's second district. The mayor wanted to make health and medical services more accessible to as many families as possible. With this project, residents of this district or those living in Payatas, Batasan Hills, Bagong Silangan, Holy Spirit and Commonwealth barangays, and those from nearby barangays are expected to benefit from the construction of a three-storey hospital building fully equipped with modern medical facilities that is scheduled to start this year. Aside from the project, the mayor also assured QC residents of an expanded access to affordable health care thru the modernization of the two city-owned hospitals – the Quezon City General Hospital (QCGH) and the Novaliches District Hospital (NDH).

17

http://www.quezoncity.gov.ph/index.php?option=com_content&view=article&id=1196:construction-of-new-quezon-city-hospital&catid=1:latest-news&Itemid=362

d) Lupong Tagamayapa at Pang-kaayusan (Peace and Order Council)

QC's Disaster Risk Reduction and Management Office

Quezon City government has formally activated the Disaster Risk Reduction and Management Office (QCDRRMO) with an initial set of officers who will perform specific functions in response to situations brought about by natural or human-induced disaster. Mayor Herbert M. Bautista has signed executive order 5 series of 2013, which established the QCDRRMO, under his direct supervision effective July 15 . The QCDRRMO was headed by Dr. Noel L. Lansang, supported by Ms. Daisy Flores for administratiion and training; Ms. Ma. Cinderella Garcia for research and planning; and Mr. Teddy Ibañez for operations and warning.The QCDRRMO two-storey building is under construction at the city hall compound facing Kalayaan Avenue. QC is the first LGU in the country with a DRRMO to be housed in its own building. The QC government has been undertaking various efforts in preparating for disasters that might occur especially this rainy season. Trainings, seminars and workshops had been conducted in different department, barangays, NGOs and NGAs to hone skills of volunteers in life saving techniques.

http://www.quezoncity.gov.ph/index.php?option=com_content&view=article&id=1185:qcs-disaster-risk-reduction-and-management-office&catid=1:latest-news&Itemid=362

6. Pakikilahok ng mga tao sa mga pagpaplano ng pamahalaang lokal sa pamamagitan ng forum

Disaster risk reduction summit for QC youth

A youth summit with the theme, “Strengthening Youth Participation in Disaster Risk Reduction and Management” was recently held at the 14th floor conference hall of the QC Hall main building. The summit aims to train and educate the young people of QC on how to respond during natural disasters such as typhoons, floods, earthquakes, and emergency situations like accidents. Speaking before summit participants comprised mostly of students from various QC high schools, Mayor Herbert M. Bautista underscored the importance of training the youth in disaster preparedness and risk mitigation skills. He said more training programs will be conducted in the future to teach the young people in other related

18

areas such as first aid training and stress handling and management. In compliance with UNICEF policies, the city government has been undertaking various efforts in disaster preparation by conducting trainings, seminars and workshops in different department, barangays, NGOs and NGAs to hone skills of volunteers, as well as citizens, in life-saving techniques. As planned, QC will also have its own geo-spatial map that can be used in city emergency response operation; mainstreaming of GIS on DRRM; integration of QC’s EM system and development of the emergency operations plan; hazard, vulnerability and risk assessment for floods and earthquake; legal and institutional arrangement, and risk sensitive land use planning. Other city officials who attended the summit included Vice Mayor Joy Belmonte and Dr. Noel Lansang, head of Quezon City Disaster Risk Reduction and Management, among others.

http://www.quezoncity.gov.ph/index.php?option=com_content&view=article&id=1237:disaster-risk-reduction-summit-for-qc-youth&catid=1&Itemid=362

II. Direktang Pakikilahok (DIRECT INVOLVEMENT)

2. Pagbibigay ng motibo ng pamahalaang lokal sa mga tao na makilahok sa mga pampublikong gawain

QC Top Taxpayers Recognized

The city government acknowledged the revenue contribution of 10 companies and 16 other business establishments in the city during the Taxpayers’ Night held at the Crowne Plaza Hotel in Ortigas last October 15, 2013. Mayor Herbert Bautista, together with Vice Mayor Belmonte and some city officials presented the plaque of recognition to the Top Ten Taxpayers of Quezon City which include Family Music Inc., SySu International Inc., The mayor mentioned in his speech that the city was able to develop more business opportunities through the help of taxes paid by these companies. He also cited the transformation of Payatas into a power producer, in addition to the planned establishment of a state-of-the-art slaughterhouse, a modern sports center and a communication center in the future. He further added that the city government is open to do business with the private sector through Public-Private Partnership (PPP) scheme. He thanked the QC business community for being the city’s partner for development and commended the QC companies who knew their social responsibility and were involved in various city government projects such as socialized housing, flood mitigation and rescue program, environment protection, and health and wellness programs, among others.

19

http://www.quezoncity.gov.ph/index.php?option=com_content&view=article&id=1233:qc-top-taxpayers-recognized&catid=1:latest-news&Itemid=362

A. Pakikilahok ng mga Mamamayan

1. Ako ay aktibong nakikilahok sa mga gawaing pampamayanan

Outstanding QC citizens, institutions honored at 2013 Gawad Parangal

The city government conferred its highest recognition awards to ten citizens and three institutions that made great contributions to the development of Quezon City and the country at the 2013 Gawad Parangal Awards night held at the Crowne Plaza Galleria on October 12. Mayor Herbert M. Bautista joined Vice Mayor Joy Belmonte, city councilors and officials, including hundreds of guests in honoring the awardees, which is part of QC’s 74th Founding Anniversary celebrations. The program was hosted by Ms. Shamcey Supsup and Marvin Cruz.

http://www.quezoncity.gov.ph/index.php?option=com_content&view=article&id=1229:outstanding-qc-citizens-institutions-honored-at-2013-gawad-parangal&catid=1:latest-news&Itemid=362

3. Bilang grupo, pinahahalagahan ng mga pinuno ang lahat ng aming kontribusyong nabuo

Best Barangays highlighted in exhibit

An exhibit is currently held at the south wing of the Quezon City Hall lobby featuring the city’s barangays that excelled in various categories: best Anti-drug Abuse Council, best Peace and Order Committee, Nutrition Council, Solid Waste Management Council, Dengue Prevention and Control, Quality Legislation, Lupong Tagapamayapa, Gender Practice and Responsive Governance, and Disaster Risk Reduction Management Council. The best barangays were chosen out of the 142 barangays from the city’s six districts based on their performance and achievements in grassroots governance.

20

Direct involvement

From the past three years, the city government of Quezon City had been conducting a lot of projects concerning participatory governance. The major projects of the city include ordinance, peace and order, environmental awareness, infrastructures, job openings, health, safety and protection of their people, and help for those affected by calamities.

The great thing about the city of Quezon is that they respect everyone and accept all kinds of people no matter what they are. They do not show discrimination and promote an anti-bullying society which has been proven in their project on preventing acts of bullying committed among students in the area. To further address this issue, the city made an ordinance creating a Protection Center for Women, Children and Lesbians, Gays, Bisexual and Transgenders (LGBT) who are victims or survivors of violence and abuse, adopting a comprehensive program thereof and for other purpose.

To further promote participatory governance in the area, the city accredited the one hundred thirty two (132) Non-Governmental Organizations (NGOs) and People’s Organizations (POs) pursuant to the provisions of the local government code of 1991 and its implementing rules and regulations, in relation with the Department of Interior and Local Government (DILG).

For the peace and order of the area, the city institutionalized the Quezon City Tripartite Industrial Peace Council (QC-TIPC).

The platform centered on further improving the following key areas/sectors: Health and Nutrition, Education and Culture, Livelihood and Economic Development, Peace and Social Security and Sustainable Good Governance.

The main thrust was on how to create Jobs and livelihood, programs that will create Opportunities for all — Youth, women and all sectors. The main strategy for the platform/agenda to be realized is through “Bayanihan sa Pamamahala”.

There were series of community-based consultation-workshops in each 24 areas of Quezon City wherein the the workshop was presented back to the people for approval, adoption and endorsement.

NGOs in Quezon City pursued wider social aims; they created projects where the community have been encouraged to participate and to contribute from the first period of planning until the implementation of the project. Implementing projects for the local people and encourage them to participate was a great practice for them to be involved in the activities and operations the organizations have for the development of the city. The NGOs prioritized community development and training in participating and contributing on projects and operations the organization would implement. This aims

21

to help local people to know their rights in receiving services or providing such and their responsibilities to act as an individual or group in achieving certain goals pertaining to the development of the city.

Facilitating seminars for communication was also a priority for it aims to help local people to be participative and be mindful on issues and aspects of social, environmental, political, and economical, etc. that affects their lives. Having knowledge and consciousness can be helpful for them to have a clear understanding. Good communication with the local people can strengthen their knowledge and involvement to the projects.

Supporting innovation projects was also a priority and was effective in building a participatory governance for it helped the city to meet the needs of the local people through technology and new ideas. The technology aids organization´s projects to be convenient and efficient in providing services to local people. The convenience and the efficiency to ensure good services encourage local people to be more productive and participative.

These are some of NGOs projects and operations that were implemented in Quezon City.

The Quezon City Council passed on third and final reading on July 20, 2009, PO 2008-111 entitled AN ORDINANCE STRENGTHENING AND INSTITUTIONALIZING THE SYSTEM OF PARTNERSHIP IN LOCAL GOVERNANCE BETWEEN THE QUEZON CITY GOVERNMENT AND THE PEOPLE OF QUEZON CITY.

The ordinance was filed by Quezon City District 3 Councilor Jorge “Bolet” Banal, Jr., and was pushed by Task Force PAT, a network of more than 200 organizations in Quezon City. CPE acts as the convener of the task force.

The measure which is also called PAT Ordinance of Quezon City, aims to establish the People’s Council of Quezon City (PCQC) which will serve as the self-regulating umbrella organization of all accredited Civil Society Organizations and business/private organizations in the City. The PCQC had a mechanism in selecting CSO/Private sector participation in the city development council, other local special bodies, task forces, and legislative committees. It aims to promote transparency in governance, accountability through effective allocation of resources and implementation of basic services through active people’s participation. The ordinance was also advocated by the Council of Sectoral Representatives (CSR), the 50 NGO-PO Representatives to the City Development Council. It was claimed by CSR as one of its major accomplishments by institutionalizing people’s participation in city governance. La Salle Institute of Governance, through Mr. Mon Padilla also took active participation in the task force.(Participation, Accountability and Transparency (PAT) Ordinance of Quezon City Passed by City Council. (2009). http://urbangov.wordpress.com/2009/09/08/participation-accountability-and-transparency-pat-ordinance-of-quezon-city-passed-by-city-council/)

Basic sectors, especially the poor, were enlightened on the importance of voting, particularly in 2010 election. The election was an arena wherein the poor has to exercise its power in putting pro-people and reform oriented public officials, both at the national and local level that may open up and widens

22

spaces for people empowerment process and development both at the national and local level. Good governance will start in having a clean and honest election– a dream that Filipinos have been dreaming for since a long time ago.(CPE Supports Quezon City Voters Information Campaign: Ibangon ang Dangal ng Halalan. (2009) http://urbangov.wordpress.com/2009/09/09/cpe-supports-quezon-voters-information-campaign-ibangon-ang-dangal-ng-halalan/)

Center for People Empowerment gave a seminar on Empowering and Participatory Governance to 30 urban poor leaders who are partners of the Claretian Urban Poor Apostolate. The participants, mostly represented urban poor associations that faced a threat of eviction and demolition once the Central Business Ditrict is started by the city government. Edwin Chavez, Executive Director of CPE gave an overview on empowering and participatory governance. He clarified that governance is about power and power should emanate from the people. Wilson Requez also presented the different avenues for people’s participation such as local development councils, local development planning, local legislation, mandatory consultation, recall, legislative and referendum among others. Atty. Jae dela Cruz of Akbayan Legal Team also discussed the salient features of Katarungang Pambarangay.

Actions LGUs (Q.C) taken in solving problems (2011)

The Mayor underscored the vital role that local government units play in providing an effective feedback mechanism as to the impact of the various programs and projects implemented by national government agencies in their respective localities. These consultations, give participants the opportunity to ventilate their concerns and harmonize their programs; in the end this will redound to the benefit of the people they serve

Vigilance

http://quezoncitycouncil.ph/index_readmore-legis-e-library.php

With the initiative to raise awareness on the legislative policies of the city, the 18th Quezon City Council through its Presiding Officer, Vice Mayor Joy G. Belmonte instituted the creation of a Legislative E-Library (Legis E-Lib) located at the Ground Floor of the Legislative Building, Quezon City Hall which will cater the needs of the various sectors of the society.

Stakeholders in the business sector, civic groups, non-government organizations and residents of Quezon City can now obtain a full copy of ordinances and resolutions they wanted to study or for their own guidance. The Legis E-Library features a comprehensive database of legislative tracking system of all the ordinances and resolutions being passed as well as the measures since 1939.

This Legis E-Library is just one of the primary actions that the Council is conducting in order to promote vigilance especially among its citizens. Through this initiative, the City Council is more assured with the knowledge and awareness of the citizens on the various policies existing in the city in building a law-abiding society towards development.

23

4.3 SAN JUAN

San Juan City Participatory Projects

2010: Maytunas Creek Clean- Up

- It is a clean up operation with the help of the MMDA, PRCC, River Patrol, DENR- SCO, LBC Couriers, Sagip-Pasig Movement and other NGO’s.

Community involvement is a very important aspect of revitalization for any community, no matter what size. Without community buy-in, a project may never get off the ground or will not be accepted once it is completed. (SMART E.org, 2010).

Citizen engagement values the right of citizens to have an informed say in the decisions that affect their lives. It emerged from the ideas of public participation, which is distinguished below from public communication and consultation. (Amanda Sheedy, 2008).

It was success because they manage to lessen the dirt in the creek.2011: “Tapat ko, Linis Ko”

- A short program was held to launch the project, an advocate for clean environment. It aims to remind the residents of San Juan to maintain the cleanliness in and outside of their residences and its surroundings.

- It was success because almost all of the people participate in that project.

2012: “San Juan Clean Up Drive”

- It aims to clean the city after the monsoon wind or habagat struck the city with force winds and rain.

- It was a success project because a resident of Brgy. San Perfecto sincerely thanked Mayor Guia Gomez “Damang dama po naming mayor ang gobyerno ng San Juan, ang bayanihan at ang pagtutulungan ng bawat isa”.

2013: “Lakad Bisikleta sa Araw ni San Juan”

- It was an environmental event organized by the City Government of San Juan. Bikers, Skateboarder enthusiast, aerobics expert showcase their talents to further spruce up and event and encourage more San Juanenos to participate and eventually venture in the said sports activities.

24

Meaningful consultation is not a structure; rather it is a process that underpins educational decision making. Meaningful consultation is necessary when decisions are made that will have an impact on a student’s educational program, and it is essential that this process includes the student’s family and/or caregivers. To the extent appropriate to the student’s age and ability, the student should also participate in the process. (BC CASE, 2008).

http://www.valenzuela.gov.ph/index.php/a

Expressing the sense of the Sangguniang Panlungsod of San Juan City, MM in supporting the observance of earth hour 2010 on March 27, Saturday at 8:30 p.m. and join the global call to action to every individual, every business and every community through the world to stand up, to take responsibility, to get involved and lead the way towards a sustainable future by turning off their lights enroute to making 2010 the world’s largest global climate change initiative ever.

(2011)

“Protect and preserve our environment.” this is the message of Mayor Guia G. Gomez to the members of the Sangguniang kabataan sa SK of the 21 barangays in the City of San Juan.

2. How is local governance performed in terms of participation of the people? To what extent is their participation/contribution?

The local government of Valenzuela created the Citizen’s Charter for the dawn of the information age has signal the rapid development of new systems in governance that needs to address people's concerns at the soonest time possible without sacrificing the quality of service and courtesy required of government employees.

With this Charter, the public is introduced to a guide on the standard of service that will afford them with the necessary information on how to proceed with any type of transaction with the City Government.

This will serve as the "Code" for the City Government employees in providing excellent service. It reminds them of the nature of their assigned tasks and the level of performance expected of them to accomplish. It is a mirror of a transparent and accountable government.

25

This also serves as a mechanism to promote people's participation in governance by letting their voices heard. As such, also contained therein are the communication channels where the public can provide comments and suggestions on the service they have been provided with. And, if it so warranted, they also encourage through the proper procedures and mediums, filing of complaints in relation to requests and applications which were unsatisfactorily served. The support and vigilance of the public are necessary to strengthen the entire system, and to improve on aspects and procedures which could have been overlooked.

3. What are roles of NGO’s in building participatory democracy? How effective is their participation and contribution?

Non-Government Organizations or known as NGO are organizations that are non-profit and whose members are voluntary citizens, which are organized on a local, national or even international level. NGO has a very important role in building democracy in our community. There are several Non-Government Organizations in the Philippines and in each group or organizes and driven around a specific issue and interest they want to fight for such as in human rights and health, NGO build and establish democracy because they are the organization between the people and the State. The NGO’s are the one who speak or work for and behalf of the poor and disadvantaged people in the community. NGO’s are organizations that are voluntary and registered or formally organized by people who aim to help those people who cannot speak for themselves. They provide analysis and expertise; serve as early warning and help monitor and implement international agreement. NGOs’ have been playing a vital role in strengthening political, social, economic efforts in the country. Because of NGO and the people behind it, a lot of poor people talked and shared issues to be solved and hopefully it has been solved with the cooperation of the community and with the help of the entire participant in that organization. By this democracy is shown and made while discrimination to people is lessened. People should fight for their rights since we are all equal. Leaders in NGO have a goal and to achieve that goal everyone should cooperate and fight for everyone’s right just like fighting for the Country’s Democracy. The Leader has an important role since they are the ones to rule and control the group. They are responsible for the right actions and decisions to be made and established. Voluntary participants are also responsible because they have the right to suggest and reject whether the thing has to be done or not. Everyone in the community have important roles that need to be accomplished. Following government rules because it is for the goodness of everyone. NGOs are organizations that are necessary because they work for the normal people who have nothing and cannot pay for them. They are advocates who serve and help people in order for peace to be known. Usually, people who join in organizations like Non-government organizations are in the normal sectors wherein they aim and fight for justice. Non- Government Organizations are important because it plays a vital role in the country wherein they strengthen the social, political awareness and justice.

26

NGOs are also essential to the participatory democracy because of the key element which are people involvement and the lifting of the bounds of rules, regulations and thus creating limitations which came from departments in the government, bureaucracies and in general, politics. The main difference in a GOs and NGOs are free to set up their own rules that in compliance with their purpose as an organization. For example is the Bantay Kalikasan by Marlo Mendoza in NCR, Philippines. Aside from government involvement, people such as ordinary citizens, investors are part of this group. They are more free with the implementation and application of their projects because of the absence of government limiting the funding and scope of application because of politics and practical issues as defined by the government, but instead their funding as any other NGOs are, comes from donations and investments therefore not in the need for government budget and restrictions but now can apply and implement their own programs.

These NGOs are the very symbol of volunteering without expecting in return. The sense of volunteering in NGOs is very similar in participatory governance such that when the government and the people work together, exchanging ideas and each one reflecting on their own progress, they can achieve to have a better democracy.

When asked how effective can NGOs be in the involvement of Participatory governance, my answer is they can be very effective in participation and contribution. It is because of the given opportunity by the government for not only the people but also the NGOs that they can now be involved in the decision making for the country to progress. When NGOs help the government in participatory democracy, they can give information as to how their different purposes of function as organizations contribute to the knowledge of the government that now they can implement programs such as outreach and infrastructure development especially to those who really need it because the people and NGOs voice had been heard. The benefits of NGOs in participatory democracy are the variations of purposes by the NGOs such as medical, relief, elder-related issues, healthcare especially in the rural areas that can help the government assess its strength to expand further the help, the funding and especially the people, and the government organizations thereby improving economy, alleviating poverty and recognizing issues and concerns not heard before.

4. What are the factors that contributed to the success or failure of building participatory governance?

Participatory Governance is important. The people have the right to know what is happening in the government. Participation is essential because it is part of democracy. Also participation can reduce corruption by making sure that the public officials are doing what they need to do. For this to work out, the government needs to be transparent to the citizens. Community participation has been known to have improved the efficiency of irrigation systems in many parts of the world by making use of local knowledge on soil conditions, water velocity and shifting water courses; of water and sanitation projects, by ensuring that these are sited where they are most likely to be used; and of public work projects, by utilizing local knowledge about safety hazards and vandalism. Participation can also improve efficiency by ensuring better monitoring and verification. Yet another way in which participation can enhance efficiency is by reducing costs and by augmenting resources in ways that are not available to

27

outsiders. Nowadays, it is rare to see the participation of the citizens to the decision making in the government. It is the people’s right to express their feelings, insights.

As Hoskins said: 'Participation in civil society, community and/or political life, characterised by mutual respect and non-violence and in accordance with human rights and democracy' (Hoskins 2006). It’s not only the government that’s responsible for our country. The citizens need to participate because every opinion matters. Ideas can make a change. Many people don’t care about Participatory governance and don’t have any interest about it. It fails because people don’t put time and effort to know more about it and how to participate and also they don’t know what it is, people lack information about it even the basic information. Participatory Governance fails because of some reasons. As it is said that they don’t have much information about it, sometimes there are other factors. Some of the citizens don’t have time for extra activities like these because of work and such. Some do not want to participate because they think that it’ll just be useless and not worth their efforts. Also some don’t trust the leaders. It fails because of such things.

Participatory Governance doesn’t always fail. There are people that work for the community. It succeeds because the leaders are good, the people feel that what they are doing will help and will not be wasted. They are well informed, well educated and they understand what participatory governance is. There are many factors that may contribute to the success or failure of participatory governance. For success active cooperation of the citizens is one of the factors because it is where the people need to be informed or aware of what is happening and also have an involvement in participating in the local government. It is where also the people to know where they stand for and to know what the benefits of the project. The other factor that contributed to the participatory government is the objectives or vision of the project/program because you will know where and how the project will go. Next is the effective of the project. It is important that the local officials must think wisely and plan in creating the project they must also considered the risk of it and also the good monitoring and control and lastly well- managed partnerships between public/private agencies and the people of the city.

Discipline is another factor in the success of the project because when the projects are implemented the people must follow the rules and they are well educated to obey and understand it and the project will be successful. Another is the sufficient funds and proper financial it is very important that the people in the city know the transparency and accountability to know where their money goes. Also the local officials must not steal the money of the people because the people are working hard to pay their taxes and some officials are just stealing their money for their own benefit. It is also one of the factor that it satisfy the needs of the citizens and timing to be able to build relationships and gain trust to the people also the impact that expected in the outcome of the project.

Another factor that may help in the success of the participatory government is that they consult to the citizens by listening to their opinions and gaining feedback about the issues. Because to know what other recommendation to improve the project. It is also important that they are open - minded of what the ideas or comments of the citizens towards the project. It is for the own good of the citizens and they will benefit on it. The local government must not be biased and do their job.

28

In failure there are also many factors in building the participatory governance. First is the lack of interest and capacity of the citizens in the local government. You must attract the people to gain interest and build strong relationship with them. Also encouragement is very important to be able to actively engaging citizens in participating and support in the project. Second is the insufficient knowledge about the rights of the people because of this they don’t know where to stand and protect themselves so some people will deceive them.

They must educate to know the rights of the citizens to be able to be active in participating in the government. Next is the poor quality of leadership and without proper direction. Being the leader you must have the quality to lead your citizens and be a good role model to them. You will guide them on the right path or direction. They vote you because they trust you and they know that you will help them but if you’re not doing your job their vote is useless and not deserving to be a leader of the city and the people will not believe or trust you anymore. You must be dedicated on your job because the citizens are expecting you good results. Also if the leader is insufficient in understanding of government process and the skill needed in relation to building good governance.

Another factor that leads to failure is the lack of law or ordinance you must follow the law and the citizens are knowledgeable and informed about the ordinance. Lack of coordination, communication and appropriate governance is also the factor of failure. Coordination and communication with the people is very important because both of parties will not understand each other. Another is the lack of cultural awareness and an understanding of indigenous communities and also the lack of skill to work in the government.

Failure and success are always occurring in building a participatory governance. It is not always the success that wins, sometimes because of this it will reflect as a leader and as a citizen to improve and to have a betterment and development in the city.

In order to achieve or become a success it must be a collaboration of active partnership and leadership of the officials in the city. It must also that the program is effective that will help the citizens like livelihood, education, health programs, infrastructure and innovation and etc. Also the government must be accountable and have a transparency so that people will be informed and aware about the government. Proper implementation of the law or ordinance is also a factor to achieve success and the citizens are knowledgeable enough to know their rights as a citizen of the city.

Failure occurs when there is no cooperation and not in good terms between the local officials and the citizens. Also when the local officials are not doing their job but instead they steal the money of the people for their own benefit.

Participatory governance is very important the citizens must be aware of their participatory regarding in the government but there are limitations on it. People must be active in participating in the local government.

SAN JUAN CITY

29

(PARTICIPATORY MECHANISMS)

1. Ipinaalam at itinuturo ng pamahalaang lokal ang mga proyekto sa publiko

2. May mataas na antas na bisyon at pagtugon ng pamahalaang lokal upang magkaroon ng tamang institusyon

SAN JUAN CITY Mission and Vision

VISION

San Juan: A City of Excellence

MISSION

1. Formulate and adpt plans and programs of the City

2. Develop human resources through accessible from local, national and international sources

3. Enact relevant legislation responsive to the needs of the City

http://www.sanjuancity.gov.ph/index.php?option=com_content&view=article&id=98&Itemid=98

3. Pagpapaunlad ng pamahalaang lokal na magkaroon ng ugnayan sa pamamahala sa pamamagitan ng mga reporma ng Local Special Bodies (LSB) tulad ng:

a) Lupong Pangkalusugan (Health Board)

We envision the City of San Juan to be a place where every resident enjoys affordable health care - such as medicines, professional medical service, hospitals and clinics - and economical nutrition.

In their commitment to better serve constituents, the city government of San Juan has adopted measures to further reach out to the poor by making the amenities of modern medicine available at

30

least possible cost. This was achieved through the construction and operation of a government hospital, the San Juan Medical Center. Located at the back of the city hall, it has a total floor area of 4,000 square meters.

The hospital has an initial capacity of 50 bes and eventually expanded to 150 after it was classified as Medical Center. The hospital is equipped with the latest technology for San Juan Populace.

San Juan has a total if nine (9) barangay health centers, one (1) barangay puericulture center, seven (7) barangay health stations, two (2) nutrition service offices - City Nutrition Office and the Street Food Training Center, Population Commission - Family Planning Office, officers. San Juan also boasts of three (3) private clinics and two (2) private hospitals. (http://www.sanjuancity.gov.ph/index.php?option=com_content&view=article&id=210&Itemid=127)

b) Lupong Pangturismo (Tourism Board)

Iloilo, San Juan City sign sisterhood agreement

THE mayors of Iloilo and San Juan City signed a sisterhood agreement to exchange skills and knowledge on good governance, environmental protection, and tourism, among others.

San Juan City Mayor Guia Gomez, along with son Senator JV Ejercito and 80 officials, visited Iloilo for a two-day orientation (August 15-16) about the city and how to win awards like Galing Pook, Seal of Good Housekeeping and National Competiveness. Gomez said her own officials can learn from the best practices, including the effective cleanup drive in Iloilo River and preservation of old heritage and houses of the people in Iloilo City.

http://www.sunstar.com.ph/iloilo/local-news/2013/08/16/iloilo-san-juan-city-sign-sisterhood-agreement-298200

c) Lupong Pampaaralan (School Board)

San Juan City wants to outlaw bullying

31

A San Juan city councilor has proposed an ordinance that required all schools in the city to come up with a concrete policy against bullying. Councilor Angelo Agcaoili said in a statement e-mailed to the Inquirer that the “San Juan City Anti-Bullying Ordinance” was in response to earlier reports of the Sangguniang Kabataan that bullying in the city’s schools has reached alarming proportions. Bullying, which he defined as “any severe or repeated use by one or more students of a written, verbal or electronic expression, or a physical act or gesture to put someone in reasonable fear,” will be prohibited in San Juan once the ordinance, which was recently passed on first reading, is approved. “Several studies showed that most students in primary and high schools have experienced various forms of abuse from their peers such as harassment or provocation, foul language, name calling, tormenting or even commenting negatively on someone’s looks, clothes and body,” he said.

d) Lupong Tagapamayapa at Pang-kaayusan (Peace and Order Council)

Improve police visibility to lessen the proliferation of crimes and maintain the low crime rate in San Juan,boost the morale of peace-keeping officials (police, fire, barangay police, traffic enforcers and other deputized SJPD,State-of-the-art equipment, transformation,Continuous training that enhances their skills, Re-echo all peace and order programs to the barangays and support their initiatives

(http://www.sanjuancity.gov.ph/index.php?option=com_content&view=article&id=104&Itemid=131)

9. Pakikilahok ng publiko sa bawat hakbang ng pagpaplano hanggang pagtutupad

II. Direktang Pakikilahok (DIRECT INVOLVEMENT)

2. Pagbibigay ng motibo ng pamahalaang lokal sa mga tao na makilahok sa mga pampublikong gawain

San Juan City goes slow on ‘zebra lane’ law

Wanting to prevent a repeat of the confusion sowed by a new ordinance banning colorum buses in Manila, the author of a newly passed law sanctioning motorists who don’t step on the brakes when pedestrians use the zebra lanes in San Juan City has called for a grace period. Councilor Angelo Agcaoili said those who violate Ordinance No. 1-2013, which was passed in April, will not yet be given tickets or sanctions. Melchor M. Robles, head of Task Force Disiplina in charge of implementing the ordinance, said that because of the grace period, more than 200 violators were merely advised about their offenses and released. Most of the violators, he said, were drivers of jeepneys and motorcycles from other cities.

http://newsinfo.inquirer.net/488983/san-juan-city-goes-slow-on-zebra-lane-law

San Juan City Participatory Projects of 2011

32

“TAPAT KO LINIS KO CAMPAIGN”

In celebration of the World Environment Day, the city of San Juan launched the posting of 3,000 “TAPAT KO, LINIS KO”, tarpaulins in the 21 barangays last January 6, 2011.

A short program was held to launch the project during the flag ceremony followed by the ceremonial posting at Julius store by Hon. Mayor Guia G. Gomez, an advocate for clean environment, This was succeeded by postings in the different barangays the city counsilors and the respective Barangay and SangguniangKabataan Councils.

Mayor Gomez said, The implementation of “TAPAT KO LINIS KO”, under Ordinance No. 34 – 2001, aims to remind the residents of San Juan to maintain the cleanliness in and outside of their residences and surroundings. The ordinance, authored by then councilor Clemente Bargas, imposes a 25- peso penalty for ther first offenders.

City Environment and Natural Resource Office (CENRO) chief Dante Santiago said Sari-Sari stores, offices, junkshops, repair shops and residential areas were the main targets of the cleanliness campaign, Santiago, a former three- term councilor, vowed to implement the other environmental-related ordinances of the city like the Anti-littering ordinance,the anti- smoking ordinance.

Lopez-Alozo, M. (2011) “TAPAT KO LINIS KO CAMPAIGN”.

The chief of the City Environment and Natural Resources office (CENRO),explained the environmental laws being enforced by the CENRO in the city,namely; The Ecological Solid Waste Management Act, Clean Water Act, Clean Air Act and the Toxic Substances and Hazardous and Nuclear Waste Act, He also mentioned the environmental city ordinances, and programs being implemented by Mayor Gomez like the “TAPAT KO, LINIS KO”program.to be re-launched as part of the observance of the World Environment Day every June 5 each year.He further encourage the young leaders to enact ordinances for the youth in their communities.

33

Critical Success Factors: Why eGovernment Projects Succeed

PARTICIPATION AND CITIZEN INVOLVEMENT

Participation and public involvement are promoted by planners, policy practitioners, and academics alike as a way to improve planning processes. Much attention has been paid to the different participatory methods and tools that are available and their strengths and weaknesses (for example, Van Asselt and Rijkens Klomp 2002, Lynam et al. 2007).

From early on, academics have distinguished between different forms of participation and have warned against a solely instrumental use of participation as a means of legitimizing previously established ends. Arnstein (1969) was critical of the lower rungs of her ladder of participation and warned about manipulation, therapy, and tokenism. According to Arnstein (1969), “real” participation could be found only on the higher rungs of her ladder. In a similar vein. A good illustration of this is given by Lawrence (2006), who uses the example of volunteer recorders who, despite being involved “only” on a low level of participation, gained a genuine sense of involvement and empowerment.

Increasingly, the academic debate about participation is moving away from questions of different kinds of participation and ways to organize them towards the actual effects of participation in practice. This orientation on practices of participation has provoked a wide array of critical studies that have investigated the intended and unintended consequences of participation, and question whether the various ideals of participation, including consensus, better decisions, legitimacy, and support are actually met. In most of these studies, such failures are attributed to participation being organized in the wrong way, for example, designed in ways that are inappropriate for the local context and issue at stake (e.g., Dietz and Stern 2008). Other studies, inspired by post-structuralist conceptions of power, take a more fundamental stance and conceive of participation as yet another technique to exert control over local inhabitants and as a new form of colonial oppression (i.e., Cooke and Kothari 2001).

These studies make a very important contribution to the scientific debate about participation because they focus the attention on how participation is much more than just a neutral place where citizens can make themselves heard and represent themselves. Conceptions of what citizens are and how they are supposed to behave are deeply implicated in how participation is organized and put into practice (Leach et al. 2005). Thus, participation influences how citizens can become involved and be represented. This is relevant in light of the objective of this article to understand the different ways in which citizens respond to participatory initiatives and the different forms of citizen involvement that are achieved by

34

participation. We use the term “citizens” not in a narrow way to refer to “the general public” but in a broad way that includes all members of a particular community as possible participants, including so-called representatives and stakeholders. The issue at stake is how individuals become involved, how they enact their citizenship in practice, and how that is affected by the context of participation.

The following overview presents different, but related, common experiences in which participation influences the ways in which citizens can become involved. All result in boundaries, restrictions, and limitations. The researcher does not present these as a complete overview of experiences with participation. However, studies of participation that have resulted in such critical reflections are underexposed in the scientific literature. More importantly though, a discussion of how participation structures and restricts citizen involvement suits the purpose of this article, which is to understand why and how participation may lead to unintended consequences and why it sometimes fails to meet its objectives. This article focuses on how participation creates citizens rather than offering a neutral space in which they can represent themselves. A discussion of these performative effects of participation is provided in the conclusion of this article.

Participation includes some and excludes other citizens

Although participation is often inspired by the desire to open up decision-making processes to wider audiences, it is clear that it is impossible to involve everybody and that representation can never by complete (O’Neill 2001). Participation is inevitably selective when it comes to who is able to participate. Some individuals recognized as relevant participants are considered to be part of the citizenry, while those excluded are left without a voice, without a way to express their involvement and enact their citizenship.

Participation represses differences and requires citizens to achieve consensus

Many participatory processes explicitly aim for consensus because this is considered to be a robust basis for high quality and legitimate decisions (Innes and Booher 2004). However, consensus comes at a cost: “it can create a false sense of closure and the illusion of stability” (Bloomfield et al. 2001:503). Mosse (2001) argues that the aim to achieve consensus easily leads to implicit pressure to exhibit conformist behavior. This can lead to the emergence of groupthink, a mode of thinking in which “the members' strivings for unanimity override their motivation to realistically appraise alternative courses of action” (Janis 1972:9). Diversity in perspectives is thus suppressed. The notion of community that is used in participation further adds to this. Participation can treat local communities as homogenous units with

35

common perspectives and interests, thereby overlooking the problematic character of defining communities and community interests (Selfa and Endter-Wada 2008) and suppressing intra-community diversity and difference (Mohan and Stokke 2000). Pre-existing commitments to achieve consensus thus influence how citizens can become involved and what they are expected to do. A risk exists that they become committed to achieving consensus and are assimilated into the process to such an extent that they loose sight of their original motivations for participating.

Participation reinforces dominant frameworks and expects citizens to accept them

In terms of problem definition and space for negotiation, participation is often restricted right from the start (e.g., Gonzalo-Turpin et al. 2008). For example, it may involve choosing between pre-developed policy alternatives or scenarios or developing solutions for predefined problems. While these existing problem definitions or alternatives often remain unquestioned, they shape participatory processes in important ways. As a result, existing dominant frameworks are reproduced and reinforced (Wagemans 2002). Because “participatory approaches and methods also serve to represent external interests as local needs, dominant interests as community concerns, and so forth” (Mosse 2001:22), powerful interests are strengthened and empowerment of participants is often not achieved (Stringer et al. 2006). Participation assimilates the participants and incorporates them into the projects of the already powerful. In this sense, participation is little more than a new technique with which, under the veil of inclusivity, existing goals can be reached in effective and newly legitimated ways (Kabeer 1996, Parfitt 2004). Thus, pre-existing definitions of problems influence citizen involvement because they structure and restrict the possible outcomes of participation and influence what citizens can and cannot do.

Participation excludes citizens without the required knowledge and skills

The extent to which meaningful participation is possible depends largely on the specific capacities and skills of the participants (Wagemans 2002). Expressing your views and representing your interests in an effective and meaningful way within dominant frameworks requires not only knowledge of these frameworks but also communicative skills, creativity, and guts. And not everybody possesses those equally: “ capacity to say and to be heard are unevenly distributed” (O’Neill 2001:484; also see Kohn 2000). Under the veil of Habermasian ideals of communicative rationality and the superiority of “the best argument”, technical, scientist, and elitist discourses often dominate participatory processes, effectively silencing those who are uncertain about their capacity to join these discussions (Goodwin 1998, Pellizzoni 2001). Participation can certainly contribute to developing those capacities (Dietz and Stern 2008). However, if this aspect is neglected, participation runs the risk of excluding those citizens who lack those capacities, rendering them unable to become involved and be heard.

36

Turnhout, E., S. Van Bommel, and N. Aarts (2010). How Participation Creates Citizens: Participatory Governance as Performative Practice . [ONLINE] Available at: http://www.ecologyandsociety.org/vol15/iss4/art26/. [Last Accessed December 4,2013].

Participating in formulating the fundamental goals as well as in planning and carrying out an activity empowers stakeholders and fosters a sense of ownership. These facilitate effective project implementation, conscientious monitoring of activities, and sustainable outcomes.

Effective poverty reduction also requires greater flexibility in responding to problems and unexpected opportunities throughout project development, implementation, and monitoring.

Responsiveness and collaboration among intended beneficiaries, government, civil society, and the private sector at local, intermediate (district, province, etc.) and national levels promote social capital development and sound governance.

Valenzuela City Government Emerges as Health Governance Champion

The local government of Valenzuela’s increase of investment for health and the City Health Office’s (CHO) geriatric program paved the way to the city’s newest milestone as one of the “Champions of Health Governance” in the country, distinguished by Merck Sharp & Dohme, the Kaya Natin! Movement for Good Governance and Ethical Leadership, and the Ateneo School of Government, at the Bayleaf, Intramuros, tonight, June 20, 2013.

The Champions for Health Governance (CHG) is an initiative recognizing top performing Local Government Units (LGUs) for excellent and efficient implementation of their local health programs on Millennium Development Goals (MDGs) 4, 5 and 6. Evaluation and screening of the health programs were based on their effectiveness, innovativeness, community engagement, local leadership, and transparency and accountability.

37

Valenzuela City’s presentation of its heath governance is anchored on the Aquino Health Agenda (AHA), through Administrative Order No. 2010-0036, known as the“Kalusugan Pangkalahatan” (KP), which seeks to ensure equitable access to quality health care by all Filipinos beginning with those in the lowest income quintiles.

Mayor Sherwin T. Gatchalian attributes the success of the city health programs not only to his efficient medical team, but also to all who believed and supported his accountability and transparency platforms. “The trust of our taxpayers needs to be repaid by quality service that can improve lives, and health governance is one its essential forms,” Mayor Win said.

Participatory Mechanisms

The City Social Welfare and Development Office (CSWDO) granted some 23 children with cash assistance of PhP 2,000 at a simple ceremony held at the New City Government Complex recently.

The turnover is part of the Education Assistance Program for Street Children implemented throughout the National Capital Region (NCR) by the Department of Social Welfare and Development (DSWD) in coordination with Local Government Units (LGUs).

(2012)

“In the aim of reducing the student-to-classroom ratio in Valenzuela City, four new school buildings costing Php 52,836,042 were constructed by the City Government at different campuses through the Local School Board (LSB).”

City Mayor Sherwin T. Gatchalian lead city officials, members of the Local School Board (LSB), officials from the DepEd-Valenzuela and members of various Parents and Teachers Associations (PTA) in the ribbon-cutting ceremony of the new facilities as part of the celebration of the 14th City Charter Day Anniversary. In a speech delivered before the audience, Mayor WIN bared plans to construct more school buildings throughout the City. “We give our utmost priority to education because it is an essential tool in shaping our city’s future through youth who are well-equipped with knowledge and confidence to face the challenges ahead,” Mayor WIN said. City Engineer’s Office Officer-in-Charge, Ms. Praceli S. Nelson said that the new school building will ease out student congestion in four primary school campuses, namely: the Coloong Elementary School, A. Deato Elementary School in Barangay Balangkas, A. Fernando Elementary School in Barangay Malanday and Rincon Elementary School.

38

Rincon Elementary School, on the other hand, was expanded with the construction of a 2-storey building with five classrooms and a total floor area of 552 square meters. With a population of 778 that is expected to rise in the following years, the building was built with provisions for expansion.

DepEd-Valenzuela Schools Division Superintendent, Dr. Flordeliza R. Mayari praised the active support of Mayor WIN to the educational needs of Valenzuelanos. She noted that the surge in building school facilities resulted in the excellent performance of Valenzuelano elementary students who bagged the No. 1 spot in the National Achievement Test (NAT) for the National Capital Region (NCR) for the last two consecutive years.

“Our school building program cannot be pursued without the strong partnership with the Local Government. In fact, more schools are scheduled to be inaugurated next month and continuously the end of the year,” she said.

Anchored on “Good Education is Good Governance” stance, this wave of classroom construction part of the Local Chief Executive’s “WIN and Edukasyon” program, a multi-stakeholder partnership to improve the quality of education in the City. One of the fundamental goals of the program is to reduce the student-to-classroom ratio to the ideal 45:1 for elementary schools and 50:1 for secondary school. (Urieta (2012 15)http://www.valenzuela.gov.ph/index.php/article/news/779)

(2013)

“Two weeks before classes start, volunteer students, teachers, parents and private individuals get their hands dirty cleaning public schools in Valenzuela City on the first day of the Department of Education’s Brigada Eskwela 2013.”

The program, also known as National School Maintenance Week, ran from May 20 to May 25. At the Valenzuela National High School (VNHS) in Barangay Marulas, the almost 200 volunteers who have participated in freshening up the 32 classrooms in the 4 school buildings also included VNHS alumni, members of civic and religious organizations, employees of private organizations and students from a local private university. Aside from lending their hands, the volunteers has also donated cleaning and carpentry materials, as well as floor tiles, wall fans, and an LCD projector, according to Florence Daco, the VNHS coordinator for Brigada Eskwela. Luisa Nuñez, president of Parent-Teacher Association (PTA) at VNHS, has been joining Brigada for three years now.

39

“Hindi lang teachers ang dapat naglilinis (ng mga paaralan),” (Cleaning up the school is not only the teachers’ work) said Nuñez, whose son is an incoming high school senior. “Obligasyon din ng mga magulang na maglinis. (Parents, too, have to do their part.)”

The support from stakeholders has helped earn VNHS the award Best School Implementor of Brigada Eskwela in Valenzuela City for three years in a row: in 2010, in 2011, and in 2012. VNHS Principal, Margarito Materum credits this achievement to his administration’s proactive approach. “We would contact everyone, write them letters,” he said, referring to stakeholders from various groups whom he tapped to help out.

2013: “Pailaw sa Barangay”

- Valenzuela City inaugurates the third batch of community beneficiaries of Pailaw sa Barangay, Dec. 13, 2013. The five communities, including C. Francisco St., Paso de Blas, Sitio Santan street in Marulas, Family Compound, Flaviano/A. De Castro street, San Vicente in Karuhatan, add up to a total of 31 areas provided by the local government with street lighting services.

(2010)

“Valenzuela City Mayor Sherwin T. Gatchalian top billed a local and international delegation of stakeholders on critical urban issues at a dialogue sponsored by the World Bank recently, highlighting the City’s experience and lessons on sustainable housing projects which underscores the multi-stakeholder partnership as an effective ingredient to a successful slum-upgrading program.

Hosted by the Philippines Urban Consortium, the knowledge-sharing event held at the Crowne Plaza Galleria Manila in Pasig City which was participated in by Local Government Units (LGUs), national government agencies, People’s Organizations (POs), Non-Government Organizations (NGOs), the academe, and foreign resource speakers from Thailand, Japan and the United States revolved around the theme “City Competitiveness and Social Inclusion: Learning from Innovations.”

Through the invitation of Christopher Pablo, World Bank’s Senior Operations Officer for Sustainable Development Department – Philippines, East Asia and the Pacific, the City Chief Executive spoke about the success story of the City Government’s flagship housing project “Disiplina Village,” to provide valuable opportunity for participants to learn more about slum improvement efforts for their respective

40

localities. A sustainable partnership between the City Government, the private and business sectors, the non-government organizations (NGOs) and the project beneficiaries themselves made Disiplina Village project a reality. Mayor WIN emphasized the commitment of the City Government to address poverty by “investing on people to mold an empowered community with responsible homeowners by making them active partners in developing their own community. Community empowerment programs,such as trainings, exchange visits to other model communities and social preparations are formulated to further help them improve their impoverished situation,” he said.

I. Mekanismo ng Pakikilahok

1. Ipinaaalam at itinuturo ng pamahalaang lokal ang mga proyekto sa publiko.

City Government Launches Inner Roads Lighting Project

Security in Valenzuela City steps up as more streetlights have been installed in selected communities .

A total of 32 streetlights have been installed by the City Engineering Office in various communities, through the local government unit’s street-lighting project dubbed as, “Pailaw sa Barangay,” a brainchild project of outgoing mayor, now Valenzuela City District 1 Representative, Cong. WIN Gatchalian.

c) Lupong Pangpaaralan

Valenzuelano Bayanihan Spirit Lives On For Brigada Eskwela

Two weeks before classes start, volunteer students, teachers, parents and private individuals get their hands dirty cleaning public schools in Valenzuela City on the first day of the Department of Education’s Brigada Eskwela 2013.

The program, also known as National School Maintenance Week, will run from May 20 to May 25.

At the Valenzuela National High School (VNHS) in Barangay Marulas, the almost 200 volunteers who have participated in freshening up the 32 classrooms in the 4 school buildings also included VNHS alumni, members of civic and religious organizations, employees of private organizations and students from a local private university.

Aside from lending their hands, the volunteers has also donated cleaning and carpentry materials, as well as floor tiles, wall fans, and an LCD projector, according to Florence Daco, the VNHS coordinator for Brigada Eskwela.

Luisa Nuñez, president of Parent-Teacher Association (PTA) at VNHS, has been joining Brigada for three years now.

“Hindi lang teachers ang dapat naglilinis (ng mga paaralan),” (Cleaning up the school is not only the teachers’ work) said Nuñez, whose son is an incoming high school senior. “Obligasyon din ng mga magulang na maglinis. (Parents, too, have to do their part.)”

41

The support from stakeholders has helped earn VNHS the award Best School Implementor of Brigada Eskwela in Valenzuela City for three years in a row: in 2010, in 2011, and in 2012.

On June 21, 2013, Ms. Linda Santiago of Valenzuela City Social Welfare and Development Office (CSWDO) and Atty. Aileen Suansing of Valenzuela City Legal Office assisted the mother of one of the victims, and Mau (name withheld), 15, a minor who was being lured to their illegal activity, and her mother, all from Barangay Karuhatan.

4. Pagkakaroon ng "citizens volunteers" upang makilahok sa mga immersion programs.

6. Pakikilahok ng mga tao sa mga pagpaplano ng pamahalaang lokal sa pamamagitan ng forum.

Valenzuela City Mayor Urges VOM Flood Control Project Construction

Valenzuela City Mayor Sherwin T. Gatchalian appeals to the authorities to reconsider constructing the cancelled Valenzuela-Obando-Meycauayan (VOM) Flood Control Project, saying, without its implementation, the flood control efforts of the city will be useless and could only worsen the situation of the neighboring areas.

Valenzuela City has several existing flood control infrastructures to ease up flooding, such as pumping stations and flood gates, but these are not enough that is why the city hired Irrigation/Flood Control Expert, Dr. Rafael Apostol to help design its own. “But if we implement our own flood control project, kapag may baha, itatapon lang namin ang tubig sa mga kalapit na lugar, sila naman ang kawawa. Hindi naman iyon tama, kaya kailangan ng multi-city approach sa pagtugon sa problema sa baha and that is the VOM project.” (Whenever there is flooding, we will just pump out the water to the neighboring areas, they are the ones who will suffer. That is not fair, that is why we need a multi-city approach to address flooding issues and that is the VOM project.)”

VOM project is the phase 2 of the Japan International Cooperation Agency (JICA)-funded CAMANAVA Mega Flood Control Project, which completion has been delayed for years. The remaining section, 1.7-kilometer long boulder dike in Dampalit, Malabon City is yet to be finished before the year ends.

CAMANAVA, which stands for Caloocan, Malabon, Navotas and Valenzuela, but Mayor WIN cleared that “Valenzuela City is not included in the more than 4-billion peso project and the phase 2 was cancelled due to lack of funding and its resurrection is being opposed by some authorities due to the perceived ineffectuality of the first phase.”

Dr. Apostol said that the government should conduct the necessary feasibility studies for the project instead of the foreign financier and its contractor. “That way, the project would be more effective,” he stressed.

42

About 15 percent of Valenzuela City, specifically, the low-lying areas in the first district always go through flooding during heavy downpour. “Valenzuelanos have long been waiting for an effective solution to the flooding and we want it now,” Mayor WIN said.

5. What are the actions taken by LGU’s in solving problems?

One of the basic actions that the Local Government Unit is to pass a law and an ordinance within the City. If you can’t get the attention of the people, maybe they have thought that through laws, people will get to understand that government is doing everything that they can do for the benefit of all. Take this as an example, environmental law, people will participate because of the consequences if they will not follow this. Sometimes, people will get to understand the meaning of government through laws and ordinances.

Second, enough education and awareness about their rights. The Government unit makes effective publicity of all the rights of the people by having assemblies. Officials, based on their own expertise, let people know their rights as citizens of the city. Example, knowing about the “Charter,” one of the projects of Valenzuela for the people. Through that charter, people will see the transparency of government officials in the city.

Poverty and occupation solution. The government focuses on giving jobs to its citizens. Maybe, they are conducting career fairs in the city in order for people to have the opportunity to get a job. Poverty is an issue not just in Valenzuela but in the whole Philippines and no one knows when and how to respond to this problem. But Valenzuela, based from others opinion, tries to seek solution on poverty and occupation problem.

Building Barangay Cooperatives. Cooperative is a way for people to collaborate according to former mayor Sherwin. Different Barangay in the City has their own Barangay where people get to collaborate and be encourage in making projects more for the whole city. Cooperatives as well unite all the families in one community, through cooperatives, proper information dissemination and wider chances of improving collaboration not just among the members of the society but as well as the government and the members of the community.

Effective leadership, transparency and accountability. People would do participate if they see honest leadership in the government that’s why the officials of Valenzuela made a “Charter” in order to promote accountability. They also have daily newspaper in order for the citizens to know what the officials are doing.

Local government of Valenzuela City provides different actions to be able to solve the problems within the municipality. The city made a Social Hall and Sangguniang Session Hall were the officials could work and have a meeting. Also they let the people and community to know where the money goes and spent. Through the help of sisterhood agreement, Valenzuela City has a strong connection with the Local

43

Government Unit to learn the "best practices" to avoid the "hindi matagumpay na practices" in leading. From the two LGUs from 2008 it becomes 35 sisterhood LGUs of Valenuzuela that stand as "Kamag-aral" in good governance.

Even the education is important because they make sure that graduates from there are highly employable and grow with confidence and good future. From the name of Mayor Win Gatchalian they came up to make a program named "WIN" were it focuses on the quality of education. Also, the funds for the education get higher to be able to give 582,772 workbooks in 5 years for free. The numbers of the students who can't study are increasing, so that Dr. Pio Valenzuela scholarship Program was made, almost 532 youth was able to grant the scholarships.

Through the negotiation between Teen Challenge Valenzuela and Teen Challenge International (One of the biggest nonprofit faith-based drug and alcohol recovery programs in America) they were able to build drug rehabilitation. Also through the helped of the program of Task Force on Out-of-School (TF-OSY) an Intervention, Reduction and Prevention stated by Mayor Sherwin Gatchalian last 2011. Many of the out school youth are now finished studies through the program.

The city is persevering to have the success by the form of innovation to increase the funds that the local government of Valenzuela earns. It also aims to increase the service in public and to maintain the trust of the community within the are. With the result of having a good economy the government provide a good education, health, peace awareness, benefits for the people and infrastructure and other projects for the Valenzuelano. Through the help of Valenzuela City charter it is easier to transact in their government. It prevent the cleanliness of the community, so the City made a anti-kalat task force last November 2011 with 10 Environmental Police Officers and & spotters that will watch and secure that the policy are follow. With the punishment a penalties the community learns to put their trashes in the proper places.

Building a "Pabahay" for those informal settlers living around Valenzuela City with the help of Gawad Kalinga Foundation, Dakilang Handog Foundation, and public sectors. There are almost 238 families now living and 226 families are still helping to build more houses. Those are the families lost their houses

44

because of bagging Ondoy. Also, National Housing Authority helps to make more Socialized Housing projects within the area that has been used by 4,691 families Valenzuelano.

Key principles

Participatory governance assessments are based on key principles:

1. Accountability: An assessment acts as a critical accountability mechanism for local communities and stakeholders with regards to the government’s performance. It helps to build the capacity of local communities and civil society organisations to monitor governance far beyond the scope of one project or programme.

2. Participation: The participation of a broad range of representatives from various arenas of society gives a more accurate perspective and important inputs based on the real situation on the ground. It also helps to build the national ownership that is needed to sustain the capacity and local pressure to monitor governance beyond a single project. In the case of REDD+, the participation of the forest communities in the process is a condition of the success of REDD+ in the long term.

3. Transparency: National actors should have unbiased access to information on the assessment process, and the results of the assessment should be made available to the public as a public good.

II. Direktang Pakikilahok

2. Pagbibigay ng motibo ng pamahalaang lokal sa mga tao na makilahok sa mga pampublikong gawain.

Tagalag Flood Drill Impresses Authorities

Representatives of city government offices who observed during the community flood drill of Barangay Tagalag on Saturday, September 28, were impressed with the orderliness with which the whole proceedings were carried out but pointed out that there were still rooms for improvement.

“I am very satisfied,” said Chief of Valenzuela City Police Police Superintendent Rhoderick Armamento. “I commend everyone for the teamwork I saw today. There was coordination and cooperation among the different units.”

The flood drill was held as part of the Partners for Resilience (PfR) program, a disaster management program for communities funded by Netherlands-based humanitarian organizations led by the Netherlands Red Cross.

Besides Tagalag, the barangays of Balangkas, Coloong, Malanday and Wawang Pulo comprise the PfR project sites in Valenzuela City.

The Tagalag drill was participated in by 175 families, or 391 individuals. Children numbered 136.

45

Armamento particularly commended Tagalag barangay captain Rene Bernardo, who, Armamento said, relentlessly monitored the mock rescue operations from the command post in Tagalag Elementary School via two-way radio.

The mock rescue operations included rescuing a pregnant woman and an elderly woman stuck in their homes, and a fisherman drowning in a fish pond.

Armamento advised barangays rescuers that they should check the vital signs of the person being rescued right away, even in the water.

The “evacuees” were housed at the Tagalag Elementary School.

Armamento also praised the Red Cross 143 volunteers, barangays health workers, and teachers for the orderly operations at the evacuation center.

3. Pagbibigay ng pamahalaang lokal ng mga hakbang upang magkaroon ng aktibong pakikilahok.

Annual Blood-Letting Campaign Draws More Participants

Valenzuela City Mayor Sherwin T. Gatchalian encouraged citizens to donate blood to the Philippine Red Cross Valenzuela City Chapter in the event dubbed as “Dugo ko, Alay ko: Birthday Bloodletting Campaign.”

The activity is part of “Araw ko, Alay ko… Valenzuelako,” a week-long celebration fêting Valenzuelano pride and identity, and their commitment to be of service to their country and fellowmen.

Sixty-five people from different walks of life – mostly employees of the City Government – contributed 450 ml of their blood to the Philippine Red Cross, Valenzuela City Chapter. A total of 28,350 ml of blood was bled during the entire event.

Adner Magat, a security guard of the Valenzuela City Convention Center, voluntarily gave 450 ml of his blood to the program, profoundly saying that his blood is not “for sale."

“We are glad that more and more people offer their blood as the years go by. Mayor WIN, a consistent blood donor, is truly leading by example,” Ms. Chongco concluded.(2011-04-05 | )

4. Pagbibigay ng pamahalaang lokal ng deliberasyon upang palakasin ang pakikilahok ng mamamayan.

Training to turn senior citizens into ‘senior netizens’

Like their grandchildren and other younger peers, some 17 senior citizens in Valenzuela City would soon be netizens, too, having completed free computer literacy training by the Telecommunications Training Institute (TTI).

46

“Even as senior citizens, we should not stop learning, as long as there are opportunities for it,” said Curabo.

Toledo said the TTI instructors taught the elderly participants in particular how to use the Internet to search for information about illnesses common among old people.

6. Ang mga pinuno ng pamahalaang lokal ay nakagawa ng sapat na proyekto para sa lipunan.

International Community Cites Mayor WIN Among Manila’s Top 40 Development Leaders

After a string of accolades for excellence in government service, an international body of aid and development professionals named Valenzuela City Mayor Sherwin T. Gatchalian as one of Manila’s top 40 “young, reform-minded, and spirited leaders” who are bringing positive impacts both here and abroad.

Dubbed as the "Oscars of Development," the Devex Manila 40 Under 40 International Development Leaders was organized by Development Executive Group (Devex) in partnership with energy firm Chevron and culminated with an awarding ceremony, February 19, 2013 at the Sky Garden of Zuellig Building in Makati City.

City Mayor Gatchalian received his award from The Honorable Social Welfare and Development Secretary Dinky Soliman and Devex president Raj Kumar.

Direct Involvement

Philippine Red Cross Valenzuela Chapter – Disaster Management Service, in coordination with the City’s Disaster Risk Reduction and Management Office and Barangay Malanday Disaster Risk Reduction and Management Council organized its biggest community drill with around 1,173 participating residents of Barangay Malanday in simulated disaster scenarios, and rescue and evacuation operations.

The whole program specifically tackled escalating levels of alerts; starting from reportage on water levels up to executing rescue and evacuation for different cases of victims, such as the elderly, pregnant and infants.

The mock evacuation center used was Andres Fernando Elementary School, utilizing 15 classrooms. The school is the same evacuation center used when Habagat struck in August 2012. Evacuees were fed and given medical check-ups, just like in real scenario.

PRC Valenzuela Stages Biggest Disaster Preparedness Drill. (2013). http://www.valenzuela.gov.ph/index.php/article/news/1454

47

Two weeks before classes start, volunteer students, teachers, parents and private individuals get their hands dirty cleaning public schools in Valenzuela City on the first day of the Department of Education’s Brigada Eskwela 2013.

The program, also known as National School Maintenance Week, will run from May 20 to May 25.

Twenty-five teams of six members each had raced against each other on a 200-meter distance across the Polo River in Barangay Poblacion during the Rescue Boat Competition held last June 22.

Community Rescuers Feted At 1st Disaster Preparedness Competition. (2013). http://www.valenzuela.gov.ph/index.php/article/news/1508

Consultation

The City Government of Valenzuela recognizes the Islamic constituents of as a vital force in the progress of the city through a coordinating and advisory body to be known as the Valenzuela Muslim Affairs Consultative Council (VMACC) in a formal ceremony held today, March 25, at the Social Hall of the New Government Complex.

Established through Executive Order No.12, Series of 2013, the VMACC shall collaborate with the local government in formulating plans, programs and policies to promote the welfare, interest and well-being of Filipino Muslims in the city. It will serve as a link between the city government, and the public and private agencies involved in such programs and projects and recommend positive actions necessary for its effective and efficient implementation.

Then Council shall be lead by City Mayor Sherwin T. Gatchalian as the Chairman, City Administrator and City Legal Officer, Atty. Teresita R. Capacillo, as Vice Chairman, and Mr. Gamal M. Bato as President, Executive Officer and Co-Vice Chairman,

It is a testimony of the city government’s efforts to include the local Muslim sector in policy formulation to further ensure their active participation in the achievement of growth goals and aspirations aimed at the promotion and enhancement of peace and order, economy, social welfare, cultural affairs and environment.

City Acknowledges Local Muslim Communities Through A Consultative Council. (2013). http://www.valenzuela.gov.ph/index.php/article/news/1423

Project budget proposals of attached government agencies of the Infrastructure Sector of Regional Development Council – National Capital Region (RDC-NCR) under the Metropolitan Manila Development

48

Authority (MMDA) underwent a rigid scrutiny during the Fiscal Year 2013 Regional Consultation and Budget Hearing hosted by Valenzuela City recently.

Highlighted on the said consultation meeting is the complementation of the projects of the national agencies to the programs of the LGUs that are embedded with the PPP (Public-Private Partnership) as an engine of growth and development for the NCR.

Vigilance

City Mayor Sherwin T. Gatchalian called for continuous active participation of sectors concerned in dengue vector containment and extermination, citing sanitation and safety precaution as the best weapons against the dreaded disease which wreak havoc during the onset of the rainy season.

The plea was made during the Anti-Dengue Summit held early this week at the Valenzuela City Social Hall, with a panel of City Health Officials, health workers, members and officers of homeowner’s associations, barangay and City officials, and members of the education sector, to discuss timely Anti-Dengue measures.

Mayor WIN admitted that there is no absolute solution in the battle against dengue. “The best way is to use every available method against it – larvae trapping and regular clean-up of our communities.”

City Health Officer, Dr. Jaime Exconde, noted that the overwhelming presence of the participants is a testament that the fight against dengue “is a serious matter everyone in the city is eager to deal with.”

(2011). http://www.valenzuela.gov.ph/index.php/article/news/510

As preparation ahead of the possible rise in the cases of dreaded disease dengue during the forecasted early onset of the rainy season when the disease is known to be at its peak, the City Health Department (CHD), in coordination with the Division of City Schools-Valenzuela City, conceptualized a simultaneous clean up-drive in all public schools in the city dubbed as “Eskwelahang Malinis, Dengue Free ang Nais! Let’s make it a habit!”

Despite the information campaigns and interventions being implemented by the CHD, and even if it is not yet the rainy season, it is very alarming that there are a few dengue cases reported in some areas of the City, specifically on the school-age group. “Dengue is a year-round peril and the school children have become the main victims of the disease. Aside from homes, we should look deeper on where they can get the disease, and the most possible place will be inside their schools,” City Health Department OIC Dr. Jaime Exconde said.

The activity was launched last March 15, with all the public elementary and high schools participating simultaneously. A program attended by CHD, DepEd and local barangay officials was held at Gen. T. de

49

Leon National High School, which incidentally was hailed by DepEd Valenzuela as the City’s Cleanest School for the first semester of the school year 2010-2011.

Valenzuela

1. Ang Hortaleza Foundation, Inc.

Address & Contact:Ang Hortaleza Social Development Center, 30-32 T. Santiago St., Canumay, Valenzuela City Phone: (02) 294-2774

Email : [email protected] Website : www.splash.com.ph

The ANG•HORTALEZA FOUNDATION, INC. (formerly SPLASH FOUNDATION, INC.) is a corporate foundation, was registered with the Securities and Exchange Commission (SEC) as a non-stock, non-profit corporation on August 6, 1997 by a group of incorporators who were interested in social developments for the advancement of Filipino progress. It is an operating organization. Total donations received for the last 2 years is P21,279,895.00. Its purposes (V-M-G), among others, are “to be known as a model social development organization working towards the advancement of Filipino progress through the promotion of self-reliance, productivity, local entrepreneurship and income generating programs”. Specifically, it aims to elevate the educational, social, economic and physical conditions of the less fortunate through community outreach programs, safeguard pediatric health through the initiation of child hospitalization assistance programs, uphold values and generate awareness through public service pursuits and provide assistance to the needy through charitable endeavors. The Foundation’s programs/projects now focus on the following areas: Enterprise Development, Educational Assistance, Health Assistance, Community Service, and Environmental Program. Geographical scope of programs: Barangay, Municipal, City and Provincial. Its sources of income include donations from sister companies (Splash Corporation, HBC, World Partners), fund raising, individuals and indirect income.

Quezon City

1. ABS-CBN Lingkod Kapamilya Foundation, Inc.

Address & Contact:

ABS-CBN Foundation Bldg., Mother Ignacia St., Brgy. South Triangle, Diliman, Quezon City

Phone: (02) 922-4842

50

Website: [email protected]

ABS-CBN Foundation, Inc. (AFI) was registered with the SEC as a non-stock, non-profit organization on July 5, 1989. It was established as a socio-civic arm of ABS-CBN Broadcasting Corporation. It is an operating Foundation. Its main office is located at Mother Ignacia Avenue corner Eugenio Lopez Sr., Drive, Brgy South Triangle, Quezon City. The foundation has a regular staff of 176, project-based staff of 217, 150 regular volunteers which can be 3,000 when needed, and 9 members of the Board of Trustees. It was certified for 5 years on Nov. 18, 2005 and has applied for renewal of certification. Total donations received of Php 415,707,855.00 for the year 2010. Its purposes, among others, are to commit significant impact in the strategic areas of child care, environment, education, disaster management by leveraging the power and reach of media and partnering with concerned sectors of the society. The Foundation?s programs now focus on the following areas: Child Protection services thru Bantay Bata 163, a 24-hour Hotline center offering a comprehensive program for the rescue and rehabilitation of victims as well as for public education on the prevention of all forms of child abuse; Environmental protection and conservation services; Educational television/radio program materials development; Disaster management, relief and rehabilitation services.. Its sources of income include fundraising activities, donations from ABS CBN Broadcasting Corporation, various donations from other corporations and individuals and interest income. They have an approved budget of Php 463,268,000.00 for the year 2011.

Bantay Bata (Child Watch) - a program geared towards the rescue and rehabilitation of disadvantaged Filipino children

Community involvement is likely to result in increased government responsiveness to the needs of constituents and the requirements for community progress. It is expected to lead to greater public satisfaction in local governance. It results in empowering private and civic actors to shoulder more responsibility for community development. Cooperation among stakeholders and achievement of meaningful impact on community development will increase the likelihood that good governance will be sustained.

Accountability clarifies responsibilities and powers needed in carrying out one’s responsibilities. Legitimacy that is established by an election or appointment is strengtjhened by an increase in community appreciation of good performance based on its understanding of who is responsible for what, and support for higher government units and the public at large. All stakeholders are accountable to the community in some way, including businessmen and community-based organizations, although elected and appointed officials have additional burdens that go hand in hand with certain privileges.

Fairness in the handling of conflicts is vital to achieving satisfactory outcomes. It is presumed that conflict management takes place within the ambit of the law. Efforts are made to reach a consensus in conflict resolution, to ensure greater acceptability by the community and reduce tensions, but when a

51

consensus is not possible, what is best for the majority and in keeping with both law and common sense should prevail.

A lot of NGO work meant charity -- relief in times of natural calamities, gift-giving at Christmas time, feeding programs for the destitute, taking care of the sick, the widowed, the orphaned. In the rural areas, it included building sanitary toilets, promoting better nutrition, hygiene and

community cleanliness, adult education focused on literacy and increasing agricultural productivity without altering tenurial systems.

The paradoxical conclusion that follows from this is that apparently, progress in a multi-stakeholder platform depends on the extent to which the invited participants deviate from what they were invited to do – that is, to represent their interests. As a result, multi-stakeholder platforms run real risks of losing touch with their constituencies, of increasing the gap between government and citizenry, and of enhancing the problem they were intended to solve. This resonates with other studies of participatory processes that have pointed to paradoxical outcomes of participation (Hajer and Kesselring 1999, Innes and Booher 2004, Chilvers and Burgess 2008, Aarts and Leeuwis 2010).

Outside the platform, meaningful participation was possible only for citizens who either had the persistence to represent their interests and express their resistance, or were creative enough to work within and around the existing frameworks and submit their own ideas and projects. The others took on a passive attitude. In the case study, we encountered two categories of passive citizens: the disappointed citizen and the disinterested citizen. Although homogenous in terms of behavior, the two categories differ with respect to the reasons and motivations for their passive behaviors. Disappointed citizens were interested in the plans for their area and in participating in the policy process but became disappointed with their opportunities for meaningful participation. Disinterested citizens were passive because they did not see the need for active participation: their problem definitions differed from those of the initiators and they did not identify with the plans or perceive them as relevant.

Passive behavior can thus emerge for radically different reasons (a point also made by McComas et al. 2006). These differences are neglected by the common conception of citizens as passive recipients of policy. Often, passive citizens are mistakenly seen as a homogenous group of people who apparently are not interested in active participation and are only require to be informed. Clearly, this view is not adequate (also see Goodwin 1998). Passive citizens could become involved in the participatory process if it is organized in a more open way – for example, if it allows for open discussions about the space for negotiation, the nature and definition of the problem, and the relevance of the plans for the inhabitants. Because the participatory processes in the -------- to do each of these, people were either not interested in participating or became frustrated.

52

The main finding of this case study is that participatory projects in the above local cities have resulted in a wide variety of responses in terms of both intended and unintended forms of community involvement. In terms of creating citizens, it may seem that the expectations of the initiators of participation have only partially been realized. They asked for interest-representing and creative citizens but got much more than they asked for.The analysis makes it clear that the unintended forms of citizen involvement were also created in the participatory processes.

In 2010 ,local government officials decided to install a multi-stakeholder platform to negotiate the design and management of city. The platform was set up first to achieve consensus about developing plans for a national park in the , and second to guide the implementation of those plans. With the aim to involve all stakeholders, government officials invited representatives of the an organization that represents the interests of the small barangays), and the tourist industry to participate in the platform. Also, outside of the multi-stakeholder platforms, participation was organized in the form of information and discussion meetings. These meetings offered an opportunity for those not included in the platform to have their say.

CONCLUSION:

In conclusion, the resources of the organizations involve voluntary provision of services by their members as well as financial support. Nevertheless, most of the organizations derive their financial resources from their members, from other individuals and from fund raising activities. However, it is worth noting that the organizations have networking and linkage with other organizations, international and local, that provide them substantial assistance for their programs, projects and activities. Thus, the organizations are without question capable of providing support by themselves or through their networks that the local government can tap. A perusal of the PPAs of the NGOs tells of the various undertakings that they provide on their own to the community in partnership with other government and non-government entities. They have shown their capabilities in implementing different social, economic and environmental programs in the municipality.

In terms of representation (Table , see page ), there is no specific type or scope of NGOs that is effective when it comes to interaction with citizens; all of them interact based on the nature of their advocacies and the frequency is activity-based or project-based. In terms of size, large groups like NGOs 3 and 4 interact more frequently with the citizens, the rest of the NGOs interact with the citizens less

53

frequently. NGOs conduct various interactions with the citizens, but in varying circumstances such as during project implementation, community outreach programs, provision of support during emergency or disasters as well as through community leaders and local chapters. However, as to frequency of their interaction with the citizens they contended that it is dependent on the frequency of the project that they implement, except for the health NGO which has its own regular radio program. Notably, none of the interactions of the NGOs with the citizens are done with LGU collaboration, which goes to show that NGOs are not involved by the LGU in the conduct of consultation for planning or implementing purposes, if there is any. Generally, the interactions of the NGOs with the citizens are based on the programs that they implement or information on advocacies that they disseminate. However, as far as consultation with the citizens as members of the Local Special Bodies, there is none. Hence, it can be observed that first, there is relatively limited contact between the NGOs and the citizens, and second, there is no clear pattern, the NGOs are not really representing the opinions of their constituency, though they may be representing their interests in a general fashion.

In the meantime, when asked about the participatory opportunities in decision-making and service delivery, some of the ordinary citizens revealed that they have not participated in the decision-making in the community and that decisions are already made before the people know because there is few consultation among ordinary citizens. In terms of service delivery, all of them have participated in their personal capacity and through collective effort of the community. Most of them however have idea on how to participate in the local government except at the barangay (village) level. Some expressed that political inclination plays an important part in participation. As to interaction with the local government in the delivery of services, most of them acknowledged the services they availed as beneficiary as well as the social and infrastructure programs that tend to help address the problems of the community. Obviously, there is a limited

referral to a particular NGO as a conduit of their participation both in decision-making and service delivery in the local government. Hence, there is lack of consultation at the ground level.

Local officials and department heads are one in saying that the local government needs the NGOs. However, the events betray the fact that the NGOs are not being tapped closely by the LGU. Despite the structure provided by the Local Government Code of 1991 for NGO participation in the Local Special Bodies there seems to be no visible or real partnership between the two sectors. Thus, the opportunities given to the NGOs have not been realized because the LGU, generally, cannot be said to be open to the idea of collaboration.

In giving importance to community involvement, the performance of cities in terms of participatory mechanisms, accountability with direct involvement, nurturing values of stakeholders ,trust and transparency with vigilance , and empowerment through consultation and collaboration, described how the local government values and maintains participatory governance. The reports on selected participatory projects which described local government communicates its operations to the public through the use of different mediums and the report on participation which described how the local

54

government attained a level of maturity in engaging the citizens and the CSOs through their participation in decision-making in the local special bodies and involvement in local development projects.

However, as it was found out, there is no effective LGU-NGO relationship and the mechanism is not effective, much less complied with. Decision-making in the local special bodies is not participatory and consultative while the citizens and NGOs barely participate in local development projects. NGOs have the skills and the resources however these are not tapped properly. Even so, the level of resources of the NGOs has little relationship to their level of participation, once a minimal level is reached.

The local officials and department heads aver that NGOs in the local special bodies are active participants but the NGOs say otherwise, while the minutes of the meetings shows very little participation. Several factors can be derived from the findings. First, the local officials and department heads are aware of the policy on NGO involvement in the local special bodies because it is provided for by law and they must follow. The partial compliance was embodied in the Executive Orders issued by the local chief executive. However, partial compliance to policy does not indicate effective participation or ensure effective involvement by the NGOs; there was no effort on the part of the local officials and department heads to reach out and involve the NGOs. They do not really care whether the NGO speaks or not or absents themselves in the meetings or planning sessions, as long as they have somehow complied with the policy. Some of the NGOs are not even aware that they are a member of a local special body. Second, NGOs get recognized only when they give or do something for the local government such as the donation of a patrol car, the conduct of medical missions, or when they volunteer on environmental programs. However, recognition does not come for basic policy-making. Planning outputs are presented for approval without NGO taking part in its formulation. Most of the NGOs maintained that they have not been invited or involved in the planning. As one NGO said, “Are we being used?” In other words, NGOs are used by the local government for their convenience; they involve the NGOs when it is convenient under the circumstances., some citizens do not feel there is partnership between the LGU and the NGOs. Further, the citizens contended that service delivery reach the intended beneficiaries when they are delivered with the NGOs and government projects are better implemented if there is NGO partnership. Some citizens went as far as saying that they prefer that government projects be implemented by NGOs because they do not trust the government.

Nevertheless, politics seems prevalent on both sides. The citizens have argued that participation in the local government depends on political inclination. In other words, if you belong to a different political party than that of the local officials you cannot participate. In the same vein, local government support comes most to those who belong to the same political party as the local officials. The citizens also took notice how NGOs deliver their services. An NGO supported by a political figure provides scholarship only to those who are politically inclined.

55

Some NGOs provide relief assistance to selected individuals especially those they know. Nevertheless, when services by the NGOs are delivered through the village leaders, only supporters of those leaders get access.

On the other hand, the disconnection between the NGOs and the citizens is largely evident. Some of them have not felt the partnership. As one citizen aptly observed, they do not come to the community to consult. Most of the citizens, furthermore, believe that the citizens are not properly represented in the LSBs.

As was shown, there is effective LGU-NGO relationship. NGOs are skilled and resources to cause effective LGU-NGOs partnership; however, it is evident that the LGU is entirely open to the idea of collaboration.

The case study exemplifies many of the criticisms of participation described earlier. The participatory processes in the case study were characterized by limited and static conceptions of citizenship. Because of the dominance of existing policies, the multi-stakeholder platforms achieved little more than their reproduction. Shifts in power relations and empowerment of the local inhabitants were not achieved. Instead, the process contributed to the reinforcement of already powerful interests. Furthermore, it is clear that meaningful participation was not easy and required specific knowledge and skills.

In an Institutional dialogue the most institutionalized type of dialogue between civil society and government occurs via the so called

‘tripartite’ institutions. Based on the model of corporatist structures in social democracies, tripartite institutions are forums where government representatives, business and trade unions come together to discuss, advise policy-makers and possibly strike deals on socio-economic policies, salaries, workers’ rights and the like (Schmitter & Lehmbruch 1979; Reutter 1996).

Such an analysis, however, neglects the active contributions of the participants and the ways in which participation is also a “resource for human agency” (Kesby 2007). Mosse (2001) and White (1996), for example, describe how the ways in which local people articulated their local needs were influenced by their expectations of what the agency could deliver. Participation thus involves not so much the representation of pre-existing local perspectives, views, or interests, but their construction in the context of participation:

Participatory practices involve context-specific interactions between participatory initiatives and the expectations they are imbued with on the one hand, and participants and the needs, identities, and views they articulate on the other.

Although it remains important to analyze exactly how participatory processes create their own participants and with what consequences for community involvement, as the researcher did in this

56

paper, it is now clear that the forms of citizen involvement identified in the case study are best regarded as articulations resulting from context-specific interactions and practices. This is consistent with contemporary ideas on citizenship as expressed, among others, by Mouffe (1993:12): “No identity is ever definitively established, there always being a certain degree of openness and ambiguity in the way [it is] articulated.” Citizenship in that way is not an apriori given but is constructed in interaction in the context of participation. Or, as Leach et al. (2005:29) put it: “citizenship [should be understood] as practised engagement through emergent social solidarities”.

A perspective of participation as performative practice goes beyond current optimistic debates about participation as a normative ideal that concentrates on the correct or incorrect application of techniques and methods. It also goes beyond the merely critical view of participation that emphasizes dominance, repression, and control. It makes it clear that participatory processes are practices that inevitably require acting, choosing, and selecting, in which expectations of participants and initiators interact, and in which forms of involvement, identities, and interests are articulated. This means that unintended consequences of participation are inevitable. Participation will always be exclusive in some way. It will come with restrictions about the scope of negotiation and who should be involved, with assumptions about the issue at stake, and with expectations about the outcome of the participatory process and what the participants should do and how they should behave. It is important to not only acknowledge this but also critically reflect on which restrictions, assumptions, and expectations are present, how they affect community involvement, and what kinds of exclusion are achieved and with what implications.

Participatory democracy is definitely not a replacement for our representative democracy. We should cherish our civil rights as well as our civil duties. Using our right to vote is one of those, and taking our personal responsibility for our local community is another one. Therefore the participatory democracy is a permanent addition , a complement to the representative democracy. Not once every four or five years, but every day of our week.

Participation of people in development programs can develop the accountability of the authority, make the program transparent and force the authority to follow the local laws. Thus, most critics describe it as essential to the sustainability of development programs (Carley 2006; Siroros 2002). However, all participation might not confirm the sustainability of development programs, unless the participation is an effective one. Here, the word ‘effective’ refers to instances when people’s participation makes a difference within the decision making processes or policy outcomes of government (Cole & Caputo 1984). The participation process is a social transformation mechanism, where the power of the implementing agency is transformed by civil society (Brett 2003). In this way ‘effective participation’ is evident when people’s empowerment reaches a position that enables cooperative and collective actions to be performed with the implementing agency, resulting in enhanced influence over decision-making, monitoring and evaluation processes (Brett 2003; Cooper, Bryer & Meek 2006).

However, some researchers have opposed the idea of people’s participation, not always seeing it as good for controlling a program towards an effective outcome. To them, participation sometimes causes delays in decision-making (Innes & Booher 2004) or increases cost (Olson 1965) or brings conflict (Bureekul 2000). Hence, some international organisations contend that participation would vary with

57

the variations of culture and socio-economic conditions of a particular society; different countries with varying socio-economic circumstances might well benefit from different models of people’s participation (OECD 2000).

The researcher have argued that the role and involvement of non government organisations (NGOs) in participatory programs in those selected cities above can bring better outcomes. Nonetheless, NGOs have been cosidered as organisers and mobilizers of urbanpeople in terms of empowerment and developing awareness for participation in group activities

The researcher is inspired to promote a reflexive view of participation as performative practice, which takes into account the intended and unintended consequences of participation in terms of inclusion and exclusion and examines the implications of these achievements for citizenship, governance, and democracy. Such a perspective recognizes that participatory initiatives will generate a variety of intended and unintended responses and considers both the initiators and the participants as actors in the sense that they act and shape participatory processes.

As the researher analysis demonstrates, organizing participatory processes is an endeavor with fundamentally unpredictable outcomes. Although such a lack of control may be uncomfortable for planners and organizers, unpredictability is also an opportunity for the emergence of new perspectives and ideas. From this perspective, unintended forms of citizen involvement are not to be discarded as solely resulting from either illegitimate repression or control or from the wrong application of a fundamentally good method. Instead, both intended and unintended responses will have to be recognized and appreciated as meaningful forms of community involvement.

58